[obm-l] RE: [obm-l] Algebra Linear - Múltiplo Escalar

2005-04-08 Por tôpico Leandro Lacorte Recova
Quando voce afirma v1=0, entao se v1 esta em R^4, 0 nao e o escalar zero e
sim o vetor nulo 0 = (0,0,0,0). Voce deveria usar outra notacao para nao
confundir.

Quando voce faz v1=0*v2, nesse caso voce usa o escalar 0 que nao e igual ao
vetor nulo 0 usado anteriormente. 

Como dizia um politico, Uma coisa e uma coisa e outra coisa e outra
coisa...

Leandro
Los Angeles, CA.


-Original Message-
From: [EMAIL PROTECTED] [mailto:[EMAIL PROTECTED] On
Behalf Of Daniel S. Braz
Sent: Friday, April 08, 2005 8:37 AM
To: OBM-L
Subject: [obm-l] Algebra Linear - Múltiplo Escalar

Problema retirado do Cap. 1.6 do livro Algebra Linear (David Lay)

Dados os vetores v1 e v2 do R4 e sabendo que nenhum dos dois é
múltiplo escalar um do outro, verifique se o conjunto formado por c =
{v1, v2} é linearmente dependente.

Eu pensei o seguinte: Já que v1 e v2 não são múltiplos o conjunto c
não pode ser L.D. Porém a resposta do livro era que o conjunto poderia
ser L.D. já que v1 ou v2 poderiam ser o vetor nulo (i.e: todas as
componentes iguais a zero). Então...minha dúvida:

O vetor nulo é considerado multiplo de todos os vetores ou de nenhum vetor?

Sendo v1 = 0 e v2 = (qq um não nulo). Se eu fizer 1*v1 = 0*v2, eu
estou dizendo que v2 é múltiplo escalar de v1? (ou seja, zero é um
escalar?)

Se zero foi escalar, então o vetor nulo não poderia ser considerado e
a resposta dada pelo livro está errada, certo?

[]s
daniel

-- 
A essência da Matemática reside na sua liberdade. (G. Cantor)

=
Instruções para entrar na lista, sair da lista e usar a lista em
http://www.mat.puc-rio.br/~nicolau/olimp/obm-l.html
=

=
Instruções para entrar na lista, sair da lista e usar a lista em
http://www.mat.puc-rio.br/~nicolau/olimp/obm-l.html
=


RE: [obm-l] matlab

2005-04-08 Por tôpico Leandro Lacorte Recova








Crie um arquivo .m e escreva:



A = zeros(10,12);



For i = 1:10

    For j = 1:12

    A(i,j)=2*i+j;

    End

End



Thats it  



Leandro



-Original Message-
From: [EMAIL PROTECTED]
[mailto:[EMAIL PROTECTED] On Behalf
Of Bruno Lima
Sent: Friday, April 08, 2005 11:31
AM
To: OBM lISTA
Subject: [obm-l] matlab





Pessoal nao to achando isso no Help , e eu to com pressa...como eu
entro no Matlab com uma matriz 10x12 tal que a(i,j)=2i+j ?









Yahoo!
Acesso Grátis: Internet rápida e grátis. Instale o discador agora!








RE: [obm-l] Conjectura de Poincare

2005-04-01 Por tôpico Leandro Lacorte Recova
Poincaré Conjecture 
  

In its original form, the Poincaré conjecture states that every simply
connected closed three-manifold is homeomorphic to the three-sphere (in a
topologist's sense) , where a three-sphere is simply a generalization of the
usual sphere to one dimension higher. More colloquially, the conjecture says
that the three-sphere is the only type of bounded three-dimensional space
possible that contains no holes. This conjecture was first proposed in 1904
by H. Poincaré  (Poincaré 1953, pp. 486 and 498), and subsequently
generalized to the conjecture that every compact n-manifold is
homotopy-equivalent to the n-sphere iff it is homeomorphic to the n-sphere.
The generalized statement reduces to the original conjecture for n = 3. 
Tirei do site: http://mathworld.wolfram.com/PoincareConjecture.html 

The Poincaré conjecture has proved a thorny problem ever since it was first
proposed, and its study has led not only to many false proofs, but also to a
deepening in the understanding of the topology of manifolds (Milnor). One of
the first incorrect proofs was due to Poincaré himself (1953, p. 370),
stated four years prior to formulation of his conjecture, and to which
Poincaré subsequently found a counterexample. In 1934, Whitehead (1962, pp.
21-50) proposed another incorrect proof, then discovered a counterexample
(the Whitehead link) to his own theorem. 

The n = 1 case of the generalized conjecture is trivial, the n = 2 case is
classical (and was known to 19th century mathematicians), n = 3 (the
original conjecture) appears to have been proved by recent work by G.
Perelman (although the proof has not yet been fully verified), n = 4 was
proved by Freedman (1982) (for which he was awarded the 1986 Fields medal),
n = 5 was demonstrated by Zeeman (1961), n = 6 was established by Stallings
(1962), and  was shown by Smale in 1961 (although Smale subsequently
extended his proof to include all ). 

The Clay Mathematics Institute included the conjecture on its list of $1
million prize problems. In April 2002, M. J. Dunwoody produced a five-page
paper that purports to prove the conjecture. However, Dunwoody's manuscript
was quickly found to be fundamentally flawed (Weisstein 2002). A much more
promising result has been reported by Perelman (2002, 2003; Robinson 2003).
Perelman's work appears to establish a more general result known as the
Thurston's geometrization conjecture, from which the Poincaré conjecture
immediately follows (Weisstein 2003). Mathematicians familiar with
Perelman's work describe it as well thought-out and expect that it will be
difficult to locate any substantial mistakes (Robinson 2003, Collins 2004).
In fact, Collins (2004) goes so far as to state, everyone expects [that]
Perelman's proof is correct.

-Original Message-
From: [EMAIL PROTECTED] [mailto:[EMAIL PROTECTED] On
Behalf Of Nicolau C. Saldanha
Sent: Thursday, March 31, 2005 6:09 AM
To: obm-l@mat.puc-rio.br
Subject: Re: [obm-l] Conjectura de Poincare

On Mon, Mar 28, 2005 at 05:58:38PM -0300, Bruno Lima wrote:
 Pessoal, uma duvida minha, ha mais ou menos ums ano
 anunciaram por ai que um russo, acho que se chamava
 Perelman havia resolvido a Conjectura de Poincare,
 depois nao ouvi mais falar, afinal resolveu?? E o cara
 recebeu o 1 mi de dolares? Pois eu acho que esse era
 um dos problemas do intituto Clay.

Tanto quanto eu sei, a demonstração ainda está sendo verificada
pelos especialistas da área, mas a impressão geral é de que está
tudo certo. A demonstração usa análise pesada. E sim, este é
um dos problemas milionários.

[]s, N.
=
Instruções para entrar na lista, sair da lista e usar a lista em
http://www.mat.puc-rio.br/~nicolau/olimp/obm-l.html
=

=
Instruções para entrar na lista, sair da lista e usar a lista em
http://www.mat.puc-rio.br/~nicolau/olimp/obm-l.html
=


RE: [obm-l] Teo. Riez

2005-03-30 Por tôpico Leandro Lacorte Recova
O livro do Reed e bem interessante ! 

O livro do Kreysig, e tambem do Rudin apresentam provas ! 



-Original Message-
From: [EMAIL PROTECTED] [mailto:[EMAIL PROTECTED] On
Behalf Of Bernardo Freitas Paulo da Costa
Sent: Wednesday, March 30, 2005 11:33 AM
To: obm-l@mat.puc-rio.br
Subject: Re: [obm-l] Teo. Riez

Bom, eu não sei se é algo que você vá gostar, mas tem o livro (na
verdade são vários, mas para você é o primeiro) Methods of Modern
Mathematical Physics, Reed  Simon, que explica bastante bem Análise
Funcional, e acho que ele prova o Teorema de Riesz, que na sua forma
geral é:
Se f(x) é um funcional linear, então f(x) = x, a para algum a e ,
é um produto interno, que por definição é uma forma bilinear simétrica
positiva definida (aqui não dá para falar de matriz, já que pode ter
base infinita!). E daí, para ter o que você quer, acho que basta fazer
uma demonstração de mudança de base.

Abraços
-- 
Bernardo Freitas Paulo da Costa


On Wed, 30 Mar 2005 14:34:22 -0300 (ART), Bruno Lima
[EMAIL PROTECTED] wrote:
 Sendo A uma matriz nxn simetrica, positiva definida entao x´Ay   (x´ é x
 transposto ) define um produto interno de x por y . Queria saber se vale a
 volta: dado um produto  ,  interno em R^n existe uma matriz A como acima
 tal que x,y=xAy 
   
 Ou seja caracteriza produto interno em R^n 
   
 Vou dar uma olhada no livro do Elon de Algebra Linear. 
 Um amigo falou pra eu olhar sobre o Teorema de Riez que sob certa
condicoes,
 caracteriza operadores lineares , achei num livro de Analise Funcional mas
 viajei um pouco, alguem sabe um bom livro onde encontro esse Teorema 
   
 Valeu, abraco
 
  
 Yahoo! Acesso Grátis: Internet rápida e grátis. Instale o discador agora! 
 


=
Instruções para entrar na lista, sair da lista e usar a lista em
http://www.mat.puc-rio.br/~nicolau/olimp/obm-l.html
=

=
Instruções para entrar na lista, sair da lista e usar a lista em
http://www.mat.puc-rio.br/~nicolau/olimp/obm-l.html
=


RE: [obm-l] sobre serie de Taylor

2005-03-29 Por tôpico Leandro Lacorte Recova
Fabio,

O Claudio ja mostrou ! A ideia basica era voce usar a regra da cadeia como
ele mostrou. 

O livro que eu me referi e aquele bem fininho editado pela Editora da UNB
nos anos 60 e depois pelo IMPA. 

Saudacoes,

Leandro

-Original Message-
From: [EMAIL PROTECTED] [mailto:[EMAIL PROTECTED] On
Behalf Of Fabio Niski
Sent: Monday, March 28, 2005 5:48 PM
To: obm-l@mat.puc-rio.br
Subject: Re: [obm-l] sobre serie de Taylor

Olá Leandro.
O Elon tem 3 livros de Analise no R^n.
Ademais, conheco a formula de Taylor para funcoes de mais de uma 
variavel, como no site do wolfram
http://mathworld.wolfram.com/TaylorSeries.html (é a formula 31)

Sei que com a serie de Taylor podemos majorar
f(x + k ,y + h) - f(x,y) por
f[x](x,y)*k + f[y](x,y)*h + O(h^2)

Mas eu não estou conseguindo identificar estes elementos na funcao phi 
em questao.

Poderia ser mais especifico por gentileza?

Obrigado!

Niski

LEANDRO L RECOVA wrote:

 O Elon tem um livro de Analise no R^n onde essa formula aparece la. Siga 
 a notacao dele e voce chega nesse resultado.
 
 From: Fabio Niski [EMAIL PROTECTED]
 Reply-To: obm-l@mat.puc-rio.br
 To: obm-l@mat.puc-rio.br
 Subject: [obm-l] sobre serie de Taylor
 Date: Mon, 28 Mar 2005 17:58:21 -0300

 Ola pessoal. Fiquei em duvida aqui em uma passagem onde foi usada a 
 serie de Taylor.

 Notacao:
 1) a[n] lê-se a índice n
 2) vou usar * para indicar multiplicacao.
 3) f[x](x,y) lê-se derivada em relacao a variavel x no ponto (x,y)

 Define-se p(x,y;h) := a[1]*f(x,y)+a[2]*f(x + p[1]*h, y + p[2]*hf(x,y))

 E ele diz que a expansao de Taylor é
 p(x,y;h) = (a[1] + a[2])*f(x,y) + a[2]*h*(p[1]*f[x](x,y) + 
 p[2]*f[y](x,y)*f(x,y)) + O(h^2)

 Gostaria que algum membro da lista por favor elucidasse esta expansão, 
 talvez deixando claro alguma passagem que o autor pulou.

 Estou tambem disponibilizando , no URL abaixo, a passagem escaneada do 
 livro (Bulirsch, Stoer) para eventuais duvidas na notação.

 http://www.niski.com/passagem.gif


 Desde já muito obrigado.

 Niski
 =
 Instruções para entrar na lista, sair da lista e usar a lista em
 http://www.mat.puc-rio.br/~nicolau/olimp/obm-l.html
 =
 
 
 
 =
 Instruções para entrar na lista, sair da lista e usar a lista em
 http://www.mat.puc-rio.br/~nicolau/olimp/obm-l.html
 =
 
 

=
Instruções para entrar na lista, sair da lista e usar a lista em
http://www.mat.puc-rio.br/~nicolau/olimp/obm-l.html
=

=
Instruções para entrar na lista, sair da lista e usar a lista em
http://www.mat.puc-rio.br/~nicolau/olimp/obm-l.html
=


[obm-l] RE: [obm-l] RE: [obm-l] Cálculo no R^n

2005-03-25 Por tôpico Leandro Lacorte Recova
Eder, eu acho que e so isso mesmo !! 

-Original Message-
From: [EMAIL PROTECTED] [mailto:[EMAIL PROTECTED] On
Behalf Of Lista OBM
Sent: Friday, March 25, 2005 1:00 PM
To: obm-l@mat.puc-rio.br
Subject: Re: [obm-l] RE: [obm-l] Cálculo no R^n

Meu caro Leandro,

minha primeira idéia foi essa, mas por achar tão
simples o problema, desconfiei dela. Por isso preferi
colocar aqui na lista pra a solução de outras pessoas.

grato, éder.
 
--- Leandro Lacorte Recova [EMAIL PROTECTED]
wrote:
 Sera que voce usando h=e_{i} onde i=1,2,…m, sao os
 vetores da base canonica
 em R^m, voce ja nao mostra a continuidade ? 
 
  
 
  
 
 Leandro.
 
  
 
 -Original Message-
 From: [EMAIL PROTECTED]
 [mailto:[EMAIL PROTECTED] On
 Behalf Of Lista OBM
 Sent: Wednesday, March 23, 2005 11:43 AM
 To: Lista OBM
 Subject: [obm-l] Cálculo no R^n
 
  
 
 Gostaria de uma ajuda no problema abaixo:
 
  
 
 Seja f: U -- R^n , U aberto de R^m, diferenciável
 numa vizinhança de um
 ponto p pertencente a U e tal que dado e = epsilon 
 0, existe d = delta  0
 tal que:
 
  
 
  || x - p ||  d == || df_x (h)
 - df_p (h) ||  e.|| h
 || .
 
  
 
 Mostre que as derivadas parciais de f são contínuas
 em p.
 
  
 
 Notação: df_x (h) é o mesmo que a diferencial de f
 em x aplicada em h (h
 estah em R^m).
 
  
 
 grato desde já, éder.
 
 __
 Converse com seus amigos em tempo real com o Yahoo!
 Messenger 
 http://br.download.yahoo.com/messenger/ 
 
 





Yahoo! Acesso Grátis - Internet rápida e grátis. 
Instale o discador agora! http://br.acesso.yahoo.com/
=
Instruções para entrar na lista, sair da lista e usar a lista em
http://www.mat.puc-rio.br/~nicolau/olimp/obm-l.html
=

=
Instruções para entrar na lista, sair da lista e usar a lista em
http://www.mat.puc-rio.br/~nicolau/olimp/obm-l.html
=


RE: [obm-l] Corpos x Fields

2005-03-17 Por tôpico Leandro Lacorte Recova
So mais uma: Field tambem pode ser traduzido como CAMPO em eletromagnetismo
(Eletric Field, Eletromagnetic Field). 

Claudio, voce esta certissimo quanto aos matematicos europeus !!! 

Leandro

-Original Message-
From: [EMAIL PROTECTED] [mailto:[EMAIL PROTECTED] On
Behalf Of Frederico Reis Marques de Brito
Sent: Thursday, March 17, 2005 12:25 PM
To: obm-l@mat.puc-rio.br
Subject: RE: [obm-l] Corpos x Fields

Meu caro amigo Cláudio, você pefgou pesado. Contra Gauss, fica difícil achar

matemático de qualquer nacionalidade. Que me perdoem Newton e Arquimedes...


Estou meio por fora do início da discussão mas FIELD tb se traduz como 
campo, VECTOR FIELDS, por exemplo. Mas em álgebra é se,mpre como CORPO.

UM abraço a todos.

From: Claudio Buffara [EMAIL PROTECTED]
Reply-To: obm-l@mat.puc-rio.br
To: obm-l@mat.puc-rio.br
Subject: [obm-l] Corpos x Fields
Date: Thu, 17 Mar 2005 15:13:57 -0300

Outra questao de jargao:

Em portugues, falamos CORPO.
Em alemao eh KORPER e em frances eh CORPS.

Por que nos paises de lingua inglesa eles falam FIELD?

Serah que acharam esquisito falar no BODY OF COMPLEX NUMBERS?

De mais a mais, nesse assunto, prefiro ficar com os alemaes e franceses.
Por que? Cite um grande matematico americano do seculo 19 (tah bom, tiveram
uns ingleses mas nao dao nem pra saida contra Gauss, Riemann, Cauchy e
Poincare...)

[]s,
Claudio.

on 17.03.05 09:32, Nicolau C. Saldanha at [EMAIL PROTECTED] wrote:

oeficientes a_j forem reais ou complexos.

 [...]
 
  Aliás, campo provavelmente é uma tradução não usual de field.
  O termo usual e correto no nosso idioma é *corpo*.
 
  []s, N.
 


=
Instruções para entrar na lista, sair da lista e usar a lista em
http://www.mat.puc-rio.br/~nicolau/olimp/obm-l.html
=

_
MSN Messenger: converse online com seus amigos .  
http://messenger.msn.com.br

=
Instruções para entrar na lista, sair da lista e usar a lista em
http://www.mat.puc-rio.br/~nicolau/olimp/obm-l.html
=

=
Instruções para entrar na lista, sair da lista e usar a lista em
http://www.mat.puc-rio.br/~nicolau/olimp/obm-l.html
=


RE: [obm-l] Aritm[etica

2005-03-09 Por tôpico Leandro Lacorte Recova








Claudio,



Acho que na segunda linha deveria ser n |
12384-9. Dai o resultado deve dar 45. 



-Original Message-
From: [EMAIL PROTECTED]
[mailto:[EMAIL PROTECTED] On Behalf
Of claudio.buffara
Sent: Wednesday, March 09, 2005
9:11 AM
To: obm-l
Subject: Re:[obm-l] Aritm[etica





Supondo que n é um inteiro positivo, teremos:





n| 7040 - 20 == n | 7020





n | 12348 - 9 == n | 12339





Logo, n | mdc(7020,12339) = 27 ==





n = 1, 3, 9 ou 27.





Mas 7020 dividido pelos 3 primeiros numeros nao pode deixar resto 20.





Logo, n = 27.











[]s,





Claudio.












 
  
  De:
  
  
  [EMAIL PROTECTED]
  
 





 
  
  Para:
  
  
  obm-l@mat.puc-rio.br
  
 







 
  
  Cópia:
  
  
  
  
 





 
  
  Data:
  
  
  Wed, 9 Mar 2005 13:20:06 -0300 (ART)
  
 







 
  
  Assunto:
  
  
  [obm-l] Aritm[etica
  
 






 



Unicamp 

Dividindo-se
7040 por n, obtem-se resto 20. Dividindo-se 12384 por n, obtem-se resto 9. Ache
n.



fiz por
tentativas, mas preferiria um método são-paulino para resolver, ou seja, mais
elegante, claro, conciso, ...








RE: [obm-l] Classes de equivalencia

2005-03-03 Por tôpico Leandro Lacorte Recova








Eu gosto do livro do Jaci Monteiro !
Agora, o IEZZI tambem tem um livro basico de algebra.







-Original Message-
From: [EMAIL PROTECTED]
[mailto:[EMAIL PROTECTED] On Behalf
Of Marcos Victor
Sent: Thursday, March 03, 2005
11:06 AM
To: obm-l@mat.puc-rio.br
Subject: [obm-l] Classes de
equivalencia





Qual um bom livro pra ver classes de equivalencia?











Obrigado.









Yahoo!
Acesso Grátis - Internet rápida e grátis. Instale o discador do Yahoo!
agora.








[obm-l] RE: [obm-l] [OFF] Livro: História da Matemática

2005-02-15 Por tôpico Leandro Lacorte Recova
Esse livro e bom ! Tem muita coisa interessante, principalmente no tempo dos
gregos, Gauss, Newton, etc...Voce vai gostar.

-Original Message-
From: [EMAIL PROTECTED] [mailto:[EMAIL PROTECTED] On
Behalf Of Bruno Soares
Sent: Monday, February 14, 2005 9:00 PM
To: obm-l@mat.puc-rio.br
Subject: [obm-l] [OFF] Livro: História da Matemática

Olá pessoal...


Vou cursar essa matéria agora e gostaria de saber se alguém tem esse
livro usado pra de vender:

HISTÓRIA DA MATEMÁTICA
Autor:  CARL BENJAMIN BOYER

Se alguem tiver entre em contato: [EMAIL PROTECTED]


Atenciosamente;


Bruno;

=
Instruções para entrar na lista, sair da lista e usar a lista em
http://www.mat.puc-rio.br/~nicolau/olimp/obm-l.html
=

=
Instruções para entrar na lista, sair da lista e usar a lista em
http://www.mat.puc-rio.br/~nicolau/olimp/obm-l.html
=


RE: [obm-l] Variedades e manifolds

2005-02-03 Por tôpico Leandro Lacorte Recova








Arthur,



Eu sou mais da area de geometria. Mas se
voce pegar um livro de Geometria Riemaniana, por exemplo, o do Manfredo, no 1o
capitulo ele ja define variedade. Existem algumas variedades interessantes por la
!! Porem o livro se concentra mais em Variedades Riemanianas. O Elon tem um
livro sobre Variedades publicado ha muitos anos atrasNa UnB tem uma
copia na biblioteca. Da um pulo por la !!! 



Leandro. 



-Original Message-
From: [EMAIL PROTECTED]
[mailto:[EMAIL PROTECTED] On Behalf
Of Bruno Lima
Sent: Thursday, February 03, 2005
4:46 AM
To: obm-l@mat.puc-rio.br
Subject: Re: [obm-l] Variedades e
manifolds





Nao sou especialista , mas...





Manifold quer dizer exatamente variedade, no sentido que as nossas maes
conhecem, tipo existe uma variedade de opcoes de sorvetes
traduzindo pro ingles esse variedade é manifold





 Intuitivamente uma variedade é um conjunto que
localmente se parece (do ponto de vista topologico, metrico) com algum espaco
euclidiano, ie, algum R^n , o tal n é dito a dimensao da variedade. Por exemplo
superficies (esfera, plano...) sao variedades de dimensao 2.





 Tem um teorema de Whitney que diz mais ou menos que
qualquer variedade pode ser colocada dentro de um certo R^n desde que esse n
seja grande o suficiente.





 Ref.: A Biblia :Elon curso de Analise
Vol. 2






Int. a variedade diferenciaveis tb do Elon.











Façlow.







Artur Costa Steiner
[EMAIL PROTECTED] wrote:






 Bom dia
 
 Eu jah vi os termos variedade e manifold, o ultimo
nao sei como se diz
 em Portugues. Jah procurei saber o que significam, mas nao encontrei uma
 referencia clara. Alguem saberia descrever sucintamente o que eles
 significam e dar alguma referência?. Creio que manifold eh
algo como um
 espoaco metrico que tem caracteristicas semelhantes aos espacos
 Euclidianos, mas esta eh uma informacao muito vaga.
 Abracos
 Artur 

=
Instruções para entrar na lista, sair da lista e usar a lista em
http://www.mat.puc-rio.br/~nicolau/olimp/obm-l.html
=









Yahoo!
Acesso Grátis - Internet rápida e grátis. Instale o discador do Yahoo!
agora.








[obm-l] RE: [obm-l] Provar desigualdade por indução

2005-01-21 Por tôpico Leandro Lacorte Recova
Porque voce nao tenta achar a soma dos n cubos naturais. Eu mandei a solucao
uma vez e a formula e:

Sn=1^3+2^3 + + n^3 = [n^2.(n+1)^2]/4 

Sn-1 = [(n-1)^2.n^2]/4 


Veja que tendo a formula, a desigualdade segue facilmente. 

Leandro


-Original Message-
From: [EMAIL PROTECTED] [mailto:[EMAIL PROTECTED] On
Behalf Of Alan Pellejero
Sent: Friday, January 21, 2005 12:18 PM
To: obm-l@mat.puc-rio.br
Subject: [obm-l] Provar desigualdade por indução

Olá a todos os amigos da lista!
Essa desigualdade é do livro do Apostol e eu não
consigo demonstrá-la.
Gostaria que alguém me ajudasse.
Grato!

1^3 + 2^3+ ... +(n-1)^3 (n^4)/4 1^3 + 2^3 + ... +
n^3

Como eu posso resolver?
Obrigado,
Alan Pellejero

__
Converse com seus amigos em tempo real com o Yahoo! Messenger 
http://br.download.yahoo.com/messenger/ 
=
Instruções para entrar na lista, sair da lista e usar a lista em
http://www.mat.puc-rio.br/~nicolau/olimp/obm-l.html
=

=
Instruções para entrar na lista, sair da lista e usar a lista em
http://www.mat.puc-rio.br/~nicolau/olimp/obm-l.html
=


RE: [obm-l] algebra linear - funcionais lineares

2005-01-13 Por tôpico Leandro Lacorte Recova








Essas demonstracoes tem no livro do Lang.
De uma olhada nesse link:



http://mathworld.wolfram.com/MatrixTrace.html




Leandro



-Original Message-
From: [EMAIL PROTECTED]
[mailto:[EMAIL PROTECTED] On Behalf
Of Lista OBM
Sent: Thursday, January 13, 2005
12:33 PM
To: Lista OBM
Subject: [obm-l] algebra linear -
funcionais lineares





gostaria de uma ajuda nos dois problemas abaixo:











1) Considere o funcioanl linear f: M_n(K) -- K definido por f(A) =
tr A (i.e., f(A) = traço de A), p/ todo A em M_n(K).











a) Mostre quematrizes semelhantes em M_n(K) têm o mesmo traço.
(Obs.: Esse naum estah muito longe de eu consegui resolve-lo.)

















b) Seja g:M_n(K) -- K um funcional linear t.q. g(AB) = g(BA), p/
todo A, B em M_n(K). Mostre queexiste b em K t.q. g(A) = b tr A, p/ todo
a em M_n(K).

















2) Seja V um K-espaço vetorialqualquer e B = {v _ j} uma base de
V (i em um conjunto de índices Jqualquer). Para cadaj em J, defina
um funcional linear f_j em V* t.q. f_ j(v_i) = delta_ij (i.e., 1 se i = j e 0
se i0). Prove que {f_ j}, j em J é uma base de V* se, e só se, J é
finito. (Obs.: A volta tem qq livro de alg. linear.)











garto desde já, éder. 





















Yahoo!
Acesso Grátis - Internet rápida e grátis. Instale o discador do Yahoo!
agora.








RE: [obm-l] oi!

2005-01-11 Por tôpico Leandro Lacorte Recova








Nao vou me estender muito. 



Excelentes empregos no exterior nao dependem
de onde voce se formou. Temos excelentes universidades no Brasil que tambem ja enviaram
alunos pro exterior. Depende muito de voce estudar e fazer um bom curso. Eu nao
fiz ITA, fiz UnB (Brasilia) e hoje trabalho ha 4 anos numa empresa de telefonia celular em Los Angeles, California. Depende muito da sua
forma de trabalho, capacidade de aprendizado e conceitos fundamentais. Claro, depende tambem
de voce ser fluente em uma lingua estrangeira. Eu tenho diversos amigos brasileiros
que trabalham aqui em Los Angeles como engenheiros e nenhum deles fez ITA ou IME. Sao excelentes escolas,
mas nao se restrinja somente a elas. PUC-Rio, UNICAMP, UnB, UFSCar, UFSC, Campina
Grande, dentre outras, sao excelentes centros. 



Leandro. 

Los Angeles, CA. 



-Original Message-
From: [EMAIL PROTECTED]
[mailto:[EMAIL PROTECTED] On Behalf
Of rafaelc.l
Sent: Tuesday, January 11, 2005
11:00 AM
To: obm-l
Subject: Re: [obm-l] oi!





Ae Kellem











Pelo que eu ouco falar dos dois institutos, acho que no IME o
ensino eh melhor, o que naum quer dizer que o aprendizadoseja melhor,
pois este depende do aluno. Mas o ITA eh historicamente mais conhecido que o
IME. o ITA tem marca de excelencia tanto aqui como no exterior, o que facilita
ainda mais as chaces de excelentes empregos para os iteanos. O IME embora seja
uma grande escola, sua fama de excelencia esta comecando a ser mostrada , coisa
que o ITA ja fez a muito tempo.











por enquanto eh isso











abraco





Rafael










[obm-l] RE: [obm-l] Variáveis complexas

2005-01-05 Por tôpico Leandro Lacorte Recova








Caro Tertuliano,



Tudo bem ? Olha, eu acho que isso sai direto
da definicao da integral de Cauchy.  Seja z0 o ponto interior a curva C e z um ponto
da fronteira. Vou denotar por INT_c a integral de linha ao longo da curva C. Entao,
como a funcao
e holomorfa, temos que f(z0) e dada por:



f(z0) = (1/2pi.i) . INT_c (f(z)/(z-z0)) dz




Portanto, tomando o modulo de f(zo) temos,




|f(z0)| = | (1/2pi.i) . INT_c (f(z)/(z-z0))
dz | 



|f(zo)| = (1/2pi) |INT_c (f(z)/z-z0)dz |  =
(1/2pi).INT_c |f(z)/z-z0| dz . Seja k = f(z) quando z pertence a C, e o fato de
que d=|z-z0| , entao,  



|f(z0)| = k(1/2pi) INT_c (dz/|z-z0|)  =
k.(1/2pi.d).INT_c dz = k.(1/2pid).L(c)  onde L( c ) = INT_C dz e o comprimento
da curva C. 





Acho que e isso. Se fiz errado alguma coisa,
por favor, me corrijam. 



(Material sobre a Integral de Cauchy para rapida
consulta: http://mathworld.wolfram.com/CauchyIntegralFormula.html
) 



Regards,



Leandro

Los Angeles, CA. 







-Original Message-
From: [EMAIL PROTECTED]
[mailto:[EMAIL PROTECTED] On Behalf
Of Tertuliano Carneiro
Sent: Wednesday, January 05, 2005
10:42 AM
To: obm-l@mat.puc-rio.br
Subject: [obm-l] Variáveis
complexas





Olá para todos! Se alguém puder me ajudar com este exercicio ficarei
muito grato.











Sejam f
uma função holomorfa numdominio U, q contem a regiaocompacta
determinada poruma curva de Jordan suave por partesC, e z um ponto
interior a essa regiao. Se k é o maximo de /f/ ao longo de C e d é a distancia
minima de z a C, entao /f(z)/ é menor ou igual
ak[L(c)/2pi.d]^(1/n),para todo n naturalnao nulo, onde L(C)
indica o comprimento da curva C.



Um
abraço!











Yahoo!
Acesso Grátis - Internet rápida e grátis. Instale o discador do Yahoo!
agora.








[obm-l] RE: [obm-l] Variáveis complexas

2005-01-05 Por tôpico Leandro Lacorte Recova








Nao vi que tinha um expoente 1^n no meu
email anterior. 



Acho que provei so para o caso n=1.  



-Original Message-
From: [EMAIL PROTECTED]
[mailto:[EMAIL PROTECTED] On Behalf
Of Tertuliano Carneiro
Sent: Wednesday, January 05, 2005
10:42 AM
To: obm-l@mat.puc-rio.br
Subject: [obm-l] Variáveis
complexas





Olá para todos! Se alguém puder me ajudar com este exercicio ficarei
muito grato.











Sejam f
uma função holomorfa numdominio U, q contem a regiaocompacta
determinada poruma curva de Jordan suave por partesC, e z um ponto
interior a essa regiao. Se k é o maximo de /f/ ao longo de C e d é a distancia
minima de z a C, entao /f(z)/ é menor ou igual
ak[L(c)/2pi.d]^(1/n),para todo n naturalnao nulo, onde L(C)
indica o comprimento da curva C.



Um
abraço!











Yahoo!
Acesso Grátis - Internet rápida e grátis. Instale o discador do Yahoo!
agora.








[obm-l] RE: [obm-l] Pra que serve a matemática?

2004-12-29 Por tôpico Leandro Lacorte Recova








Davidson,



Na verdade, a gente tem que dizer pros alunos
que aquilo sera uma ferramenta de uso em outras materias. 



Por exemplo, quando estudamos as equacoes
de Maxwell variantes no tempo, usa-se praticamente a notacao de numeros complexos.
Processamento digital de sinais usa demais a transformada-Z, transformada de wavelts, o que envolve muito
numero complexo ai no meio. 



Leandro



-Original Message-
From: [EMAIL PROTECTED]
[mailto:[EMAIL PROTECTED] On Behalf
Of Davidson Lima
Sent: Wednesday, December 29, 2004
3:17 PM
To: obm-l@mat.puc-rio.br
Subject: Re: [obm-l] Pra que serve
a matemtica?




Acho bastante vlida a questo no nosso amigo Bruno. Frequentemente os meus
alunos se deparam com perguntas do tipo: ... e onde  que eu vou utilizar
isso na prtica Acoteceu recentemente com uma explicao de conjugado
de um nmero complexo. Explicao prtica, no consegui. Apenas disse que isso
iria ajudar na diviso de nmeros complexos, o que de fato no  algo do nosso
cotidiano (pelo menos  o que eu acho).


At breve!


Davidson Estanislau


--- saulo bastos [EMAIL PROTECTED] wrote:

From: saulo bastos [EMAIL PROTECTED]
Date: Wed, 29 Dec 2004 18:10:31 +
To: obm-l@mat.puc-rio.br
Subject: [obm-l] RE: [obm-l] Pra que serve a matemtica?

Eu acho que quem faz uma pergunta dessas nao deveria estar inscrito nesta 
lista?
Sem rancores, saulo.

From: Bruno Soares [EMAIL PROTECTED]
Reply-To: obm-l@mat.puc-rio.br
To: obm-l@mat.puc-rio.br
Subject: [obm-l] Pra que serve a matemtica?
Date: Tue, 28 Dec 2004 16:43:54 -0200

Boa tarde

Pra que serve a matemtica?

Pergunta um tanto bvia, mas quando pensamos que algo  muito bvio, 
quando no estamos pensando.










The top resources for math --- http://www.Math.com/






Instrus para entrar na lista, sair da lista e usar a lista em
http://www.mat.puc-rio.br/~nicolau/olimp/obm-l.html


[obm-l] =?iso-8859-1?Q?RE:_=5Bobm-l=5D_Pra_que_serve_a_matem=E1tica=3F?=

2004-12-28 Por tôpico Leandro Lacorte Recova
Carissimo Bruno,

Essa pergunta seria o mesmo que Pra que serve tomar agua ? . Agua e algo
essencial para a sobrevivencia de todos seres humanos, assim como a
matematica e essencial para o desenvolvimento da humanidade. Energia que
chega na sua casa, Telefones Celulares, Robos em fabricas, maquinas de
tomografia computadorizada, e outras coisas. 

Seria impossivel listar toda a utilidade dessa ciencia num simples e-mail. 

Faca voce mesmo uma pesquisa no Google. 

Regards,

Leandro
Los Angeles, CA. 

-Original Message-
From: [EMAIL PROTECTED] [mailto:[EMAIL PROTECTED] On
Behalf Of Bruno Soares
Sent: Tuesday, December 28, 2004 10:44 AM
To: obm-l@mat.puc-rio.br
Subject: [obm-l] Pra que serve a matemática?

Boa tarde

Pra que serve a matemática?

Pergunta um tanto óbvia, mas quando pensamos que algo é muito óbvio, é
quando não estamos pensando.

=
Instruções para entrar na lista, sair da lista e usar a lista em
http://www.mat.puc-rio.br/~nicolau/olimp/obm-l.html
=

=
Instruções para entrar na lista, sair da lista e usar a lista em
http://www.mat.puc-rio.br/~nicolau/olimp/obm-l.html
=


RE: [obm-l] Tensores...

2004-12-10 Por tôpico Leandro Lacorte Recova









Leandro,



Eu estudei tensors em Geometria Riemaniana
mas o enfoque e bem matematico. Caso voce queira ir nessa direcao, o Elon Lima escreveu
um livro muito bom sobre tensores.



Now, se voce quer algo mais aplicado a fisica,
tem um livro do SCHUTZ chamado MODERN DIFFERENTIAL GEOMETRY e um chamado A
FIRST COURSE IN RELATIVITY. O primeiro seria melhor. 



Agora, tambem, se voce quer praticar a matematica
dos tensores e fazer muitos exercicios, tem um livro da colecao SCHAUM do
Murray Spiegel chamado VECTOR ANALYSIS and Tensor Analysis, algo assim. 



Se voce for no site www.amazon.com vai encontrar diversos livros
de tensores tambem. Eu, particularmente, gostei do livro do Elon (edicao bem antiga)
e do primeiro do Schutz. O da colecao Schaum e mais pra praticar a matematica envolvida
nos exercicios.



Leandro



-Original Message-
From: [EMAIL PROTECTED]
[mailto:[EMAIL PROTECTED] On Behalf
Of Leandro Morila
Sent: Friday, December 10, 2004
4:56 AM
To: obm-l
Subject: [obm-l] Tensores...





Olá,











Estudei um pouco sobre tensores, e suas aplicações,mas
é dificil encontrar algo na Internet sobre o assunto, gostaria de saber se
poderiam me ajudar.











Desde já agradeço,











Leandro
















[obm-l] RE: [obm-l] Álgebra linear aplicada

2004-11-19 Por tôpico Leandro Lacorte Recova
O livro do Elon tambem e um otimo ponto de partida. 

-Original Message-
From: [EMAIL PROTECTED] [mailto:[EMAIL PROTECTED] On
Behalf Of Fabio Niski
Sent: Friday, November 19, 2004 8:34 AM
To: [EMAIL PROTECTED]
Subject: Re: [obm-l] Álgebra linear aplicada


 A propósito, alguém conhece um bom livro de álgebra linear voltado mais
para
 o lado abstrato (uma álgebra linear apresentada sob o ponto de vista das
 transformações lineares, por exemplo).

Hoffman e Kunze
=
Instruções para entrar na lista, sair da lista e usar a lista em
http://www.mat.puc-rio.br/~nicolau/olimp/obm-l.html
=

=
Instruções para entrar na lista, sair da lista e usar a lista em
http://www.mat.puc-rio.br/~nicolau/olimp/obm-l.html
=


[obm-l] RE: [obm-l] [OFF-TOPIC] Universidades - Curso de Matemática

2004-10-29 Por tôpico Leandro Lacorte Recova
O curso da PUC-RJ e otimo ! Pergunte ao Nicolau ! 

Leandro. 

-Original Message-
From: [EMAIL PROTECTED] [mailto:[EMAIL PROTECTED] On
Behalf Of Daniel S. Braz
Sent: Friday, October 29, 2004 6:34 AM
To: OBM-L
Subject: [obm-l] [OFF-TOPIC] Universidades - Curso de Matemática

Pessoal,

Desculpem o off-topic..mas..eu estou para entrar na universidade
agora..e pretendo cursar matemática..porém estou meio perdido..não sei
como escolher um bom curso..alguém teria alguma dica??
ou então..se já conhecerem uma boa universidade no rio de janeiro e
quiserem indicar eu agradeço...

obrigado!

daniel.

-- 
Uma das coisas notáveis acerca do comportamento do Universo é que ele
parece fundamentar-se na Matemática num grau totalmente
extraordinário. Quanto mais profundamente entramos nas leis da
Natureza, mais parece que o mundo físico quase se evapora e ficamos
com a Matemática. Quanto mais profundamente entendemos a Natureza,
mais somos conduzidos para dentro desse mundo da Matemática e de
conceitos matemáticos. (Roger Penrose)

=
Instruções para entrar na lista, sair da lista e usar a lista em
http://www.mat.puc-rio.br/~nicolau/olimp/obm-l.html
=

=
Instruções para entrar na lista, sair da lista e usar a lista em
http://www.mat.puc-rio.br/~nicolau/olimp/obm-l.html
=


[obm-l] RE: [obm-l] superfície

2004-10-22 Por tôpico Leandro Lacorte Recova








Tente 



X=cos(at)

Y=sin(at)

Z=at 



Para a diferente de zero.  (Cilindro)



Leandro. 



-Original Message-
From: [EMAIL PROTECTED]
[mailto:[EMAIL PROTECTED] On Behalf
Of Lista OBM
Sent: Friday, October
 22, 2004 10:14 AM
To: [EMAIL PROTECTED]
Subject: [obm-l] superfície





qual seria uma parametrização para a superfície regular S={(x,y,z) em
R^3 ; x^2 + y^2 = 1}?











grato, éder.









Yahoo!
Acesso Grátis - Internet rápida e grátis. Instale o discador agora!








[obm-l] RE: [obm-l] RE: [obm-l] superfície

2004-10-22 Por tôpico Leandro Lacorte Recova
Leo,

Valeu pela correcao  Estava distraido. 

Leandro

-Original Message-
From: [EMAIL PROTECTED] [mailto:[EMAIL PROTECTED] On
Behalf Of Leonardo Paulo Maia
Sent: Friday, October 22, 2004 11:11 AM
To: [EMAIL PROTECTED]
Subject: Re: [obm-l] RE: [obm-l] superfície

Leandro, essa parametrizacão que você deu descreve uma curva (hélice) em R^3
(ela pertence ao cilindro dado, mas é uma curva), e não uma superfície, que
requer dois parâmetros, s e t. Uma parametrizacão adequada à superfície dada
é

x=cos(t)
y=sen(t)
z=s

Leo

Quoting Leandro Lacorte Recova [EMAIL PROTECTED]:

 Tente 
 
  
 
 X=cos(at)
 
 Y=sin(at)
 
 Z=at 
 
  
 
 Para a diferente de zero.  (Cilindro)
 
  
 
 Leandro. 
 
  
 
 -Original Message-
 From: [EMAIL PROTECTED] [mailto:[EMAIL PROTECTED] On
 Behalf Of Lista OBM
 Sent: Friday, October 22, 2004 10:14 AM
 To: [EMAIL PROTECTED]
 Subject: [obm-l] superfície
 
  
 
 qual seria uma parametrização para a superfície regular S={(x,y,z) em R^3
;
 x^2 + y^2 = 1}?
 
  
 
 grato, éder.
 
   _  
 
 Yahoo! http://br.rd.yahoo.com/mail/taglines/*http:/br.acesso.yahoo.com/
 Acesso Grátis - Internet rápida e grátis. Instale o discador agora!
 
 


=
Instruções para entrar na lista, sair da lista e usar a lista em
http://www.mat.puc-rio.br/~nicolau/olimp/obm-l.html
=

=
Instruções para entrar na lista, sair da lista e usar a lista em
http://www.mat.puc-rio.br/~nicolau/olimp/obm-l.html
=


RE: [obm-l] [obm-1] Probabilidade

2004-10-12 Por tôpico Leandro Lacorte Recova
Nao seria mais facil calcular a probabilidade dela nao cortar nenhuma das
faixas e usar o fato de que P(cortar)=1 - P(nao cortar) ? 

Leandro

-Original Message-
From: [EMAIL PROTECTED] [mailto:[EMAIL PROTECTED] On
Behalf Of Edward Elric
Sent: Tuesday, October 12, 2004 3:35 PM
To: [EMAIL PROTECTED]
Subject: [obm-l] [obm-1] Probabilidade

Eis um problema de probabilidade que me parece de um nivel consideravel:

Considere uma área plana, dividida em faixas de larguras iguais, a, por 
retas paralelas. Lance sobre a regiao, ao acaso, uma agulha de comprimento 
2r, com 2ra. Qual a probabilidade de que a agulha corte umas das paralelas?

Eu nao consegui, seria bom uma ajuda :)

_
MSN Hotmail, o maior webmail do Brasil.  http://www.hotmail.com

=
Instruções para entrar na lista, sair da lista e usar a lista em
http://www.mat.puc-rio.br/~nicolau/olimp/obm-l.html
=

=
Instruções para entrar na lista, sair da lista e usar a lista em
http://www.mat.puc-rio.br/~nicolau/olimp/obm-l.html
=


[obm-l] RE: [obm-l] Re: [obm-l] Re: [obm-l] Equações trigonométricas [3 problemas]

2004-10-08 Por tôpico Leandro Lacorte Recova
Ok. 

Mas se o exercicio fosse: Calcule a soma

D = 1+cos(x)+cos(2x)++cos(nx) ?  

E se fosse 

D = 1 + sin(x)+sin(2x) + ... + sin(nx) ? 

Saudacoes,

Leandro. 

-Original Message-
From: [EMAIL PROTECTED] [mailto:[EMAIL PROTECTED] On
Behalf Of Marcio Cohen
Sent: Thursday, October 07, 2004 6:34 PM
To: [EMAIL PROTECTED]
Subject: [obm-l] Re: [obm-l] Re: [obm-l] Equações trigonométricas [3
problemas]

A prova do Edward me parece estar perfeita. Ele não usou hora alguma o
que queria provar. Apenas demonstrou um resultado obviamente equivalente ao
pedido (como ele mesmo mencionou).
[]s
Marcio

- Original Message - 
From: LEANDRO L RECOVA [EMAIL PROTECTED]
To: [EMAIL PROTECTED]
Sent: Thursday, October 07, 2004 9:42 PM
Subject: RE: [obm-l] Re: [obm-l] Equações trigonométricas [3 problemas]


 Eu nao concordo com sua solucao ! Voce ja partiu do resultado que queremos
 demonstrar. O resultado e verdadeiro e voce so fez provar a igualdade.

 A ideia e a seguinte:

 a) Substitua cos(kx)=[exp(ikx)+(exp(-ikx)]/2
 b) Entao, agrupe em duas somas:

 S = (1/2) + S1 + S2,

 S1 = [exp(ix)+exp(i2x)+...+exp(inx)]/2
 S2 = [exp(-ix)+exp(-i2x)+...+exp(-inx)]/2

 c) Use a formula da soma de uma serie geometrica para S1 e S2.

 d) Fazendo umas breves manipulacoes chega ao resultado.

 Se nao conseguir, me avise, que eu mando a solucao completa para a lista.




 From: Edward Elric [EMAIL PROTECTED]
 Reply-To: [EMAIL PROTECTED]
 To: [EMAIL PROTECTED]
 Subject: [obm-l] Re: [obm-l] Equações trigonométricas [3 problemas]
 Date: Thu, 07 Oct 2004 23:20:53 +
 
 Vamos lá, primeiro vamos fazer a segunda:
 2) Mostre que:
 D=1/2 + cos(x)+cos(2x)+cos(3x)++cos(nx)= sen[x(2n+1)/2] / 2*sen(x/2)
 Essa igualdade é valida se, e somente se, 2*sen(x/2)*( 1/2 +
 cos(x)+cos(2x)+cos(3x)++cos(nx))= sen[x(2n+1)/2].
 Assim: D= 2*sen(x/2)*( 1/2 + cos(x)+cos(2x)+cos(3x)++cos(nx)) =
 sen(x/2) + 2*sen(x/2)*cos(x) + 2*sen(x/2)*cos(2x) ++
 2*sen(x/2)*cos(nx).
 Note que 2*sen(x/2)*cos(kx)= sen((x/2)*(2k+1)) - sen((x/2)*(2k-1))
 (utilizando a formula de produto em soma). Assim temos:
 D= sen(x/2) + sen((x/2)*3) - sen((x/2)) + sen((x/2)*5) - sen((x/2)*3) +
 sen((x/2)*7) - sen((x/2)*5) + ... + sen((x/2)*(2n+1)) - sen((x/2)*(2n-1))
 Fazendo as devidas simplificaçoes temos: D= sen((x/2)*(2n+1)), como
 queriamos demontrar.
 
 Agora vamos ao primeiro problema:
 1) sabendo que D= sen1º*sen3º*sen5º.sen87º*sen89º = 2^(-n) determine
o
 valor de 2n
 Note que sen(89)=cos(1), sen(87)= cos(3), sen(85)= cos(5), sen(83)=
 cos(7),..., sen(47)=cos(43).
 Olhando para o produto D, de forma diferente temos:
 D= sen(45)*[sen(1)*sen(89)]*[sen(3)*sen(87)]*...[sen(43)*sen(47)]=
 sen(45)*[sen(1)*cos(1))]*[sen(3)*cos(3)]*...[sen(43)*cos43]
 Sabemos que sen(2x)= 2*sen(x)*cos(x), logo:
 D= (2^(-22))*sen(45)*sen(2)*sen(6)*sen(10)*...*sen(46)
 Porem sen(45)*sen(2)*sen(6)*sen(10)*...*sen(46) nao pode ser trivialmente
 calculado... e mesmo que pudesse ser calculado facilmente e ele nao seria
 potencia de 2, logo o enunciado deve estar errado.
 
 _
 MSN Messenger: converse com os seus amigos online.
 http://messenger.msn.com.br
 
 =
 Instruções para entrar na lista, sair da lista e usar a lista em
 http://www.mat.puc-rio.br/~nicolau/olimp/obm-l.html
 =


 =
 Instruções para entrar na lista, sair da lista e usar a lista em
 http://www.mat.puc-rio.br/~nicolau/olimp/obm-l.html
 =




=
Instruções para entrar na lista, sair da lista e usar a lista em
http://www.mat.puc-rio.br/~nicolau/olimp/obm-l.html
=

=
Instruções para entrar na lista, sair da lista e usar a lista em
http://www.mat.puc-rio.br/~nicolau/olimp/obm-l.html
=


RE: [obm-l] ITA.....

2004-10-04 Por tôpico Leandro Lacorte Recova
Primeiro, portugues: ENFRENTAR e nao INFRENTAR ! 

Uma boa ideia, seria voce refazer todas as provas antigas e manter a
mentalidade de ir pra prova pra acertar o maximo, ou mesmo, gabaritar. 

Existem diversas referencias boas como os livros do Morgado, Iezzi, etc. 

Regards,

Leandro

-Original Message-
From: [EMAIL PROTECTED] [mailto:[EMAIL PROTECTED] On
Behalf Of Lucas Lucas
Sent: Monday, October 04, 2004 12:48 PM
To: [EMAIL PROTECTED]
Subject: [obm-l] ITA.

Bom dia meu nome é Lucas, e eu gostaria de saber quais as recomendações para

quem quer infrentar um vestibular de alto nivel como o do ITA, se possivel 
gostaria também de algumas referencias, tanto na área da matemática quanto 
das outras matérias.

Atenciosamente:
Lucas F. B.

_
MSN Hotmail, o maior webmail do Brasil.  http://www.hotmail.com

=
Instruções para entrar na lista, sair da lista e usar a lista em
http://www.mat.puc-rio.br/~nicolau/olimp/obm-l.html
=

=
Instruções para entrar na lista, sair da lista e usar a lista em
http://www.mat.puc-rio.br/~nicolau/olimp/obm-l.html
=


RE: [obm-l] sutileza, o retorno

2004-10-01 Por tôpico Leandro Lacorte Recova
Olha so, para n=3, 

0 = 0

0 = 0 + 0 + 0 

0 = (1-1) + (1-1) + (1-1) 

0 = 1 + (-1+1) + (-1+1) -1 

Isso e ainda zero.

Vamos ver para um numero par. Seja N=4, 

0 = 0 
0 = 0 + 0 + 0 + 0
0 = (1-1) + (-1-1) + (-1+1) + (-1-1)

0 = 1 + (-1+1) + (-1+1) + (-1+1) -1 

Ainda zero. 

Generalizando, para n parcelas 0, deveriamos ter 1 parcelas 1 , 1 parcelas
-1 e (n-1) parcelas (-1+1). Assim, 


0 = 1 + (-1+1) + (-1+1) + .+ (-1+1) -1 

O erro seu esta em colocar o primeiro sinal negativo e nao ter colocado a
parcela -1 no final da expressao. O agrupamento que voce fez nao esta
certo. Basta ver os exemplos que fiz pra 3 e 4 parcelas acima. 

Leandro
Los Angeles, CA. 



-Original Message-
From: [EMAIL PROTECTED] [mailto:[EMAIL PROTECTED] On
Behalf Of eritotutor
Sent: Friday, October 01, 2004 12:44 PM
To: obm-l
Subject: [obm-l] sutileza, o retorno

Desculpe-me doutor, 
Eu soh fiz uma pergunta, caso não queira responde-la, 
fique a vontade, mas não perder a polidez, afinal isso 
eh uma lista de discussão, lembra?
A palavra sutileza eh apenas um icone, na verdade 
gostaria de saber onde estah o erro na demonstraçao 
abaixo e o seu argumento nao estah correto.

 Qual a sua definicao de sutileza?
 
 From: eritotutor [EMAIL PROTECTED]
 
 0 = 0
 0 = 0 + 0 + ... + 0
 0 = (1-1) + (1-1) + ... + (1-1)
 0 = 1-1+1-1+1-1...+1-1
 0 = 1 - (-1+1) + (-1+1) + ... + (-1+1)
 
 Esse primeiro '1' vc tirou de onde?
 Nao precisa responder pq menores participam
 da lista tambem.
 
 0 = 1 + 0 + 0 + ... + 0
 0 = 1
 
 Justifique o erro que estah nessa sutileza.
 
 Considere o paradoxo de Godel:
 Suponha que Deus existe. Se Deus existe então ele 
pode
 todas as coisas. Então peça a Deus para construir uma
 pedra que ele não pode carregar.
 Explique porque esse argumento não prova que Deus nao
 existe.
 
 
 
 
 
 
___
___
 Acabe com aquelas janelinhas que pulam na sua tela.
 AntiPop-up UOL - É grátis!
 http://antipopup.uol.com.br/
 
 
 
 
===
==
 Instruções para entrar na lista, sair da lista e 
usar a lista em
 http://www.mat.puc-rio.br/~nicolau/olimp/obm-l.html
 
===
==
 
 

_
 Express yourself instantly with MSN Messenger! 
Download today - it's FREE! 
 http://messenger.msn.click-
url.com/go/onm00200471ave/direct/01/
 
 

=
 Instruções para entrar na lista, sair da lista e usar 
a lista em
 http://www.mat.puc-rio.br/~nicolau/olimp/obm-l.html
 

=
 
 
__
Acabe com aquelas janelinhas que pulam na sua tela.
AntiPop-up UOL - É grátis!
http://antipopup.uol.com.br/



=
Instruções para entrar na lista, sair da lista e usar a lista em
http://www.mat.puc-rio.br/~nicolau/olimp/obm-l.html
=

=
Instruções para entrar na lista, sair da lista e usar a lista em
http://www.mat.puc-rio.br/~nicolau/olimp/obm-l.html
=


RE: [obm-l] curvas em R^3 (geom. diferencial)

2004-09-30 Por tôpico Leandro Lacorte Recova








Questao 1:



Faca a expansao de f(t) em serie de Taylor em torno de t=0, 



    f(t) = f(0) + f(0)t + f(0)t^2/2!
+  



Note que a partir do enunciado temos f(t),v=0
o que implica (derivando em relacao a t e usando o fato que v e um vetor fixo
de R^3) f(t),v=0, f(t),v=0,,etc,
para todo t em J.   Aplicando o produto interno em ambos os lados da equacao acima,



    f(t),v =
f(0),v + tf(0),v + t^2/2! (f(0),v) + 
 = 0.  





Portanto f(t) e ortogonal a v para todo t em
J. 





-Original Message-
From: [EMAIL PROTECTED]
[mailto:[EMAIL PROTECTED] On Behalf
Of Lista OBM
Sent: Thursday, September
 30, 2004 3:40 AM
To: [EMAIL PROTECTED]
Subject: RE: [obm-l] curvas em R^3
(geom. diferencial)





De fato a parte final da questão estah com o enunciado errado. Trocar Prove que f(t) é
ortogonal a f´(t) para todo t em J.por Prove que f(t) é ortogonal
av para todo t em J.











Grato desde já, Éder.
























1) Sejam f:J -- R^3 uma curva parametrizada e v um vetor
fixado de em R^3. Suponha que v é ortogonal a f´(t)e af(0) para
todo t emJ.Prove qe f(t) é ortogonal a f´(t) para todo t em J.











2) Seja f: J -- R^3 uma curva parametrizada, com f´(t)0
para todo t em J. Prove que | f(t) | = cte não nula = f(t) é ortogonal
a f´(t) para todo t em J.











Grato desdes já, Éder Lopes da Silva.





__
Do You Yahoo!?
Tired of spam? Yahoo! Mail has the best spam protection around 
http://mail.yahoo.com 









Yahoo!
Acesso Grátis - Internet rápida e grátis. Instale o discador agora!








[obm-l] RE: [obm-l] Teoria dos Números - Ensino Médio

2004-09-29 Por tôpico Leandro Lacorte Recova
O Nicolau tem um livro no site dele bem interessante. 

Tem um livro lancado pela editora da UnB do professor Hemar Godinho e
Shrokanian sobre Teoria dos Numeros. De uma olhada la ! 

Leandro 

-Original Message-
From: [EMAIL PROTECTED] [mailto:[EMAIL PROTECTED] On
Behalf Of [EMAIL PROTECTED]
Sent: Wednesday, September 29, 2004 3:40 AM
To: [EMAIL PROTECTED]
Subject: [obm-l] Teoria dos Números - Ensino Médio





Pessoal,

Bom dia,

Gostaria de dizer que sou novo nesta lista e estou apreciando bastante as
mensagens trocadas entre os colegas.

Estou precisando encontrar uma bibliografia adequada sobre Teoria dos
Números, porém com uma linguagem acessível para alunos do Ensino Médio.

Alguém poderia me ajudar ?

Muito obrigado.

Douglas Felipe Rodrigues da Silva
Process Development Engineer - Commercial Jets
Embraer - São José dos Campos
Phone: +55 12 3927 1244 - Cell.: +55 12 9703 7257

Graduated in Electrical Engineering at POLI-USP (2002)

Under Master Degree Program on TIM (Technology and Innovation Management)
at ITA (S.J. Campos)


This message is intended solely for the use of its addressee and may
contain privileged  or confidential information.
If you are not the addressee you should not distribute, copy  or file this
message.
In this case, please notify the sender and destroy its contents
immediately.

Esta mensagem é para uso exclusivo de seu destinatário e pode conter
informações privilegiadas e confidenciais.
Se você não é o destinatário não deve distribuir, copiar ou arquivar a
mensagem.
Neste caso, por favor, notifique o remetente da mesma e destrua
imediatamente a mensagem.


=
Instruções para entrar na lista, sair da lista e usar a lista em
http://www.mat.puc-rio.br/~nicolau/olimp/obm-l.html
=

=
Instruções para entrar na lista, sair da lista e usar a lista em
http://www.mat.puc-rio.br/~nicolau/olimp/obm-l.html
=


RE: [obm-l] curvas em R^3 (geom. diferencial)

2004-09-29 Por tôpico Leandro Lacorte Recova








Questao 2) 



ð
Seja t em J. Entao,  |f(t)|=k
implica em  



|f(t)|^2
= k^2 = f(t),f(t)=k^2  



Derive a
ultima equacao em relação a t, 



2f(t),f(t)
=0  = f(t),f(t) = 0  = f(t) é ortogonal  a f(t) para
todo t em J.   (, denota o produto interno em R^3) 





    =  A volta é imediata. 




Questão 1: 



O enunciado está correto ??? Pode conferir
???  





-Original Message-
From: [EMAIL PROTECTED]
[mailto:[EMAIL PROTECTED] On Behalf
Of Lista OBM
Sent: Wednesday, September 29,
2004 8:54 AM
To: [EMAIL PROTECTED]
Subject: [obm-l] curvas em R^3
(geom. diferencial)







Gostaria que alguém me ajudasse com os exercícios abaixo:











1) Sejam f:J -- R^3 uma curva parametrizada e v um vetor
fixado de em R^3. Suponha que v é ortogonal a f´(t)e af(0) para
todo t emJ.Prove qe f(t) é ortogonal a f´(t) para todo t em J.











2) Seja f: J -- R^3 uma curva parametrizada, com f´(t)0
para todo t em J. Prove que | f(t) | = cte não nula = f(t) é ortogonal
a f´(t) para todo t em J.











Grato desdes já, Éder Lopes da Silva.





__
Do You Yahoo!?
Tired of spam? Yahoo! Mail has the best spam protection around 
http://mail.yahoo.com 








RE: [obm-l] SISTEMA

2004-09-27 Por tôpico Leandro Lacorte Recova








Tive uma ideia:



Da segunda equacao, isole b^3. Entao temos:



  b^3 = 3a^2 + 2   (1) 



Na segunda equacao, isole b^2 



    a^3 + 11 = 3ab^2 



Multiplique por b ambos os lados,



    b(a^3+11)=3ab^3 

    

Eleve ao cubo ambos os lados pra tirar o
radical



    ((a^3+11)^3)b^3 = 27a^3.b^9 

    

    (a^9+33a^2+363a+121)(3a^2+2) =
27(a^3)(3a^2+2)^3 

    

    (a^9+33a^2+363a+121) = 27(a^3)(3a^2+2)^2



    (a^9+33a^2+363a+121) = 27(a^3)(9a^4
+ 12a + 4) 



    a^9 + 33a^2 + 363a + 121 =
243a^7 + 324a^4 + 108a^3 



    a^9-243a^7-324a^4-108a^3 + 33a^2 + 363a + 121 = 0 



Usando o MATLAB, as raizes sao as seguintes:




A1 = 15.5913   , A2 =  -15.5858  , A3 =  0.7021
+ 1.1182i , A4 =  0.7021 - 1.1182i, A5=  0.9360 , A6=  -1.1558  , A7=  -0.4229
+ 0.7671i, A8=  -0.4229 - 0.7671i,

A9 =  -0.3441



A partir dai, e so substituir esses valores
em (1) e encontrar o correspondente valor de b. 





    

-Original Message-
From: [EMAIL PROTECTED]
[mailto:[EMAIL PROTECTED] On Behalf
Of samanta
Sent: Monday,
 September 27, 2004 12:31 PM
To: [EMAIL PROTECTED]
Subject: [obm-l] SISTEMA







Olá amigos,





Existe solução para esse sistema?











a^3 - 3a(b^2) = -11





3(a^2) - b^3 = -2











Grata,





Samanta





__
Do You Yahoo!?
Tired of spam? Yahoo! Mail has the best spam protection around 
http://mail.yahoo.com 








[obm-l] RE: [obm-l] Livro de Análise

2004-09-24 Por tôpico Leandro Lacorte Recova
Tem um livro de analise do Rudin. 

-Original Message-
From: [EMAIL PROTECTED] [mailto:[EMAIL PROTECTED] On
Behalf Of Jerry Eduardo
Sent: Friday, September 24, 2004 2:59 PM
To: [EMAIL PROTECTED]
Subject: [obm-l] Livro de Análise

Alguém pode me indicar um livro para o curso de Análise, sem ser o do prof.
Elon L. Lima (Curso de Análise, vol.1)?

Grato,

Jerry

=
Instruções para entrar na lista, sair da lista e usar a lista em
http://www.mat.puc-rio.br/~nicolau/olimp/obm-l.html
=

=
Instruções para entrar na lista, sair da lista e usar a lista em
http://www.mat.puc-rio.br/~nicolau/olimp/obm-l.html
=


RE: [obm-l] Funcao exponencial - logaritmos

2004-09-17 Por tôpico Leandro Lacorte Recova
Acho que o objetivo da questao era esse mesmo !! Usar logaritmo. 

-Original Message-
From: [EMAIL PROTECTED] [mailto:[EMAIL PROTECTED] On
Behalf Of Daniel S. Braz
Sent: Friday, September 17, 2004 9:39 AM
To: OBM-L
Subject: [obm-l] Funcao exponencial - logaritmos

Pessoal,

Alguem saberia resolver esta questão SEM o uso de Logaritmos??

ITA 93
Um acidente foi presenciado por 1/65 da população de Votuporanga (SP).
O número de pessoas que soube do acontecimento t horas após, é dado
por: f(t) = B/[1+C.e^(-kt)]
onde B é a população da cidade. Sabendo que 1/9 da população soube do
acidente 3 horas após, então calcule o tempo que passou até que 1/5 da
população soubesse da notícia

f(t) = B/[1+C.e^(-kt)]

f(0) = B/[1+C.e^(-0k)] = B/65

   65 = 1+C
   C = 64

f(3) = B/[1+64.e^(-3k)] = B/9

   9 = 1+64.e^(-3k)
   8 = 64.e^(-3k)
   1/8 = e^(-3k)
   2^(-3) = e^k(-3)
   e^k = 2 -- Só consigo sair com usando log :(

[]s
daniel

-- 
Uma das coisas notáveis acerca do comportamento do Universo é que ele
parece fundamentar-se na Matemática num grau totalmente
extraordinário. Quanto mais profundamente entramos nas leis da
Natureza, mais parece que o mundo físico quase se evapora e ficamos
com a Matemática. Quanto mais profundamente entendemos a Natureza,
mais somos conduzidos para dentro desse mundo da Matemática e de
conceitos matemáticos. (Roger Penrose)

=
Instruções para entrar na lista, sair da lista e usar a lista em
http://www.mat.puc-rio.br/~nicolau/olimp/obm-l.html
=

=
Instruções para entrar na lista, sair da lista e usar a lista em
http://www.mat.puc-rio.br/~nicolau/olimp/obm-l.html
=


[obm-l] RE: [obm-l] RES: [obm-l] curva soluçao

2004-09-17 Por tôpico Leandro Lacorte Recova
De uma olhada sobre catenaria nesse link:

http://www.ime.usp.br/~martha/mat2453-2002/catenaria/


-Original Message-
From: [EMAIL PROTECTED] [mailto:[EMAIL PROTECTED] On
Behalf Of Wellington
Sent: Friday, September 17, 2004 2:16 PM
To: [EMAIL PROTECTED]
Subject: [obm-l] RES: [obm-l] curva soluçao
Importance: High

Se tomarmos, por exemplo, a curva denotada por f(x)=1 ?

-Mensagem original-
De: [EMAIL PROTECTED] [mailto:[EMAIL PROTECTED] Em
nome de eritotutor
Enviada em: Friday, September 17, 2004 5:41 PM
Para: obm-l
Assunto: [obm-l] curva soluçao

Boa noite, 

Gostaria que saber se a catenaria eh a unica curva, que 
soluciona o seguinte problema:

Qual a curva que liga dois pontos fixos A e B tal que 
o valor numerico do comprimento dessa curva eh 
numericamente igual a area abaixo da curva.

[]s


 

__
Acabe com aquelas janelinhas que pulam na sua tela.
AntiPop-up UOL - É grátis!
http://antipopup.uol.com.br/




=
Instruções para entrar na lista, sair da lista e usar a lista em
http://www.mat.puc-rio.br/~nicolau/olimp/obm-l.html

=

---
Incoming mail is certified Virus Free.
Checked by AVG anti-virus system (http://www.grisoft.com).
Version: 6.0.766 / Virus Database: 513 - Release Date: 9/17/2004
 

---
Outgoing mail is certified Virus Free.
Checked by AVG anti-virus system (http://www.grisoft.com).
Version: 6.0.766 / Virus Database: 513 - Release Date: 9/17/2004
 


=
Instruções para entrar na lista, sair da lista e usar a lista em
http://www.mat.puc-rio.br/~nicolau/olimp/obm-l.html
=

=
Instruções para entrar na lista, sair da lista e usar a lista em
http://www.mat.puc-rio.br/~nicolau/olimp/obm-l.html
=


[obm-l] RE: [obm-l] curva soluçao

2004-09-17 Por tôpico Leandro Lacorte Recova
Alem disso, a catenaria gera uma superficie de revolucao chamada CATENOIDE,
que por sinal e uma superficie minima. (Curvatura media = 0). 

Regards

Leandro
Los Angeles, CA

-Original Message-
From: [EMAIL PROTECTED] [mailto:[EMAIL PROTECTED] On
Behalf Of eritotutor
Sent: Friday, September 17, 2004 4:03 PM
To: obm-l
Subject: [obm-l] curva soluçao

-- Início da mensagem original ---

  De: [EMAIL PROTECTED]
Para: [EMAIL PROTECTED]
  Cc: 
Data: Fri, 17 Sep 2004 18:16:25 -0300
 Assunto: [obm-l] RES: [obm-l] curva soluçao

Se tomarmos, por exemplo, a curva denotada por f(x)=1 ?

-Mensagem original-
De: [EMAIL PROTECTED] [mailto:owner-obm-
[EMAIL PROTECTED] Em
nome de eritotutor
Enviada em: Friday, September 17, 2004 5:41 PM
Para: obm-l
Assunto: [obm-l] curva soluçao

Boa noite, 

Gostaria que saber se a catenaria eh a unica curva, que 
soluciona o seguinte problema:

Qual a curva que liga dois pontos fixos A e B tal que 
o valor numerico do comprimento dessa curva eh 
numericamente igual a area abaixo da curva.

[]s

Realmente Wellington,  
A funçao constante e a catenaria satisfazem, 
mas sera que existem outras, e como determina-las?
Ainda sobre a catenaria vale acrescentar que catena 
significa corrente em latim, nome proposto por 
Leibnitz. Ela pode ser encontrada no ovo, na fiação 
eletrica, no calcanhar do ser humano, nas cupulas de 
catedrais antigas (catenoide), ... e eh baseado nela 
que empresas de iogurte tem desenvolvido seus produtos.

[]s


 
__
Acabe com aquelas janelinhas que pulam na sua tela.
AntiPop-up UOL - É grátis!
http://antipopup.uol.com.br/



=
Instruções para entrar na lista, sair da lista e usar a lista em
http://www.mat.puc-rio.br/~nicolau/olimp/obm-l.html
=

=
Instruções para entrar na lista, sair da lista e usar a lista em
http://www.mat.puc-rio.br/~nicolau/olimp/obm-l.html
=


RE: [obm-l] Teoria dos Grafos [Ciclo Hamiltoniano]

2004-09-16 Por tôpico Leandro Lacorte Recova








http://www.google.com/search?hl=enie=UTF-8q=Algoritmo+de+Ciclo+HamiltonianobtnG=Google+Search




-Original Message-
From: [EMAIL PROTECTED]
[mailto:[EMAIL PROTECTED] On Behalf
Of carlos augusto
Sent: Thursday, September 16, 2004
3:29 PM
To: obm
Subject: [obm-l] Teoria dos Grafos
[Ciclo Hamiltoniano]





Sou estudante do 4º periodo de ciência da computação emeu
professordeteoria dos grafos passouum trabalho em que pedia
um algoritmopara encontrar ciclo hamiltoniano. Será que alguem poderia me
ajudar! Estou precisando de algum algoritmo que seja eficiente.











Obrigado pela Atenção





Carlos Augusto.









Yahoo!
Messenger 6.0 - jogos, emoticons sonoros e muita diversão. Instale agora!








[obm-l] RE: [obm-l] Função inversa

2004-09-13 Por tôpico Leandro Lacorte Recova
Verifique se a funcao e bijetora ou nao. 

Regards,

Leandro. 

-Original Message-
From: [EMAIL PROTECTED] [mailto:[EMAIL PROTECTED] On
Behalf Of [EMAIL PROTECTED]
Sent: Monday, September 13, 2004 7:30 AM
To: [EMAIL PROTECTED]
Subject: [obm-l] Função inversa

Olá pessoal da lista boa tarde.

Como é que eu faço para saber se uma a função y = x + 3e^x é inversível ? E
sendo inversível, como faço para saber (calcular) qual é a inversa dela ?

Valeu um abraço, Marcelo.

---
iBestMail, agora com POP3/SMTP e 120MB de espaço!
Experimente: http://www.ibestmail.com.br
=
Instruções para entrar na lista, sair da lista e usar a lista em
http://www.mat.puc-rio.br/~nicolau/olimp/obm-l.html
=

=
Instruções para entrar na lista, sair da lista e usar a lista em
http://www.mat.puc-rio.br/~nicolau/olimp/obm-l.html
=


RE: [obm-l] compactos

2004-09-09 Por tôpico Leandro Lacorte Recova
Nao vou dar detalhes, mas so uma ideia de como pode ser feito. Vou assumir
que A e B sao subconjuntos de R^{n}. Se voce quiser fazer pra qualquer
espaco metrico, tente adaptar a notacao. 


AxB e fechado: Sugestao=

Acho que se voce usar as funcoes de projecao de f: AxB - A e g: AxB- B e
usando o fato de que elas sao continuas, e o teorema: 

Seja W um subconjunto fechado de A. Entao, f: AxB-A  e uma funcao continua
se e somente se a imagem inversa de W e fechado.  Chame a imagem inverse de
W de f^(-1)(W).


Seja W um subconjunto fechado de B. Entao, g: AxB-B  e uma funcao continua
se e somente se a imagem inversa de W e fechado.  Chame a imagem inversa de
W de g^(-1)(W).


Observe que f^(-1)(W) U g^(-1)(W) = AxB e lembre que a uniao de fechados e
fechado. Portanto, AxB e fechado. 


Depois, resta mostrar que AxB e limitado. Tome z=(x,y) um ponto de AxB.
Entao, devemos mostrar que existe uma constante c tal que 

|z| =  c para todo z em AxB. 

Para isso, podemos usar a norma da soma. Seja, x um ponto de A e y um ponto
de B. Como A e B sao compactos, portanto limitados, temos que existem
constantes c1 e c2 tais que: 

|x| = c1  (Norma da soma: x=(x1,x2,...,xn) = |x|=|x1| + |x2| + .. + |xn|)

|y| = c2   (Norma da Soma : y=(y1,y2,...,yn) = |y| = |y1| + ... +
|yn|)


Portanto, |z|=|(x,y)| = |x| + |y| = c1 +c2 = c, onde usei a norma da Soma
em A , B e AxB.  

Se tiver algum erro, podem me corrigir. Eu nem fiz nada no papel porque
estou num restaurante agora e escrevi meio rapido. 

Regards,

Leandro
Los Angeles, CA. 


-Original Message-
From: [EMAIL PROTECTED] [mailto:[EMAIL PROTECTED] On
Behalf Of eritotutor
Sent: Thursday, September 09, 2004 10:12 AM
To: obm-l
Subject: [obm-l] compactos

Boa tarde, 

Gostaria, por favor, da soluçao do seguinte:

Prove que o produto cartesiano de dois conjuntos 
compactos eh compacto.

Obrigado


 
__
Acabe com aquelas janelinhas que pulam na sua tela.
AntiPop-up UOL - É grátis!
http://antipopup.uol.com.br/



=
Instruções para entrar na lista, sair da lista e usar a lista em
http://www.mat.puc-rio.br/~nicolau/olimp/obm-l.html
=

=
Instruções para entrar na lista, sair da lista e usar a lista em
http://www.mat.puc-rio.br/~nicolau/olimp/obm-l.html
=


RE: [obm-l] Algebra Linear - Operadores Lineares

2004-09-09 Por tôpico Leandro Lacorte Recova
Let T be a linear operator on the finite-dimensional inner product space V,
and suppose T is both positive and unitary. Prove T = I.

Solution:


Seja T* o operador adjunto de T. Entao, dados x,y em V temos Tx,y=x,T*y


Portanto, como T e positivo, temos 0  Tx,x = x,T*x 

Como T e unitario, temos TT*=I, ou seja, T*=T^(-1) (Operador inverso de T). 

Voltando na equacao temos,

0  Tx,x=x,T*y=x,T^(-1)x = Isso implica que T=T^(-1). Logo, 

TT^(-1)=I = T^2=I = T=I. 


Leandro. 







=
Instruções para entrar na lista, sair da lista e usar a lista em
http://www.mat.puc-rio.br/~nicolau/olimp/obm-l.html
=


RE: [obm-l] Integral (Provar que...) parte 1

2004-09-09 Por tôpico Leandro Lacorte Recova
Caso nao interpretei errado a questao, 

F(x)=ln(3x)-ln(2x)=ln(3x/2x)=ln(3/2) que e constante no intervalo de 0 a
+inf.

Leandro

-Original Message-
From: [EMAIL PROTECTED] [mailto:[EMAIL PROTECTED] On
Behalf Of [EMAIL PROTECTED]
Sent: Thursday, September 09, 2004 2:05 PM
To: [EMAIL PROTECTED]
Subject: [obm-l] Integral (Provar que...) parte 1

Olá pessoal da lista, boa noite.

Estou enviando três questões sobre a matéria integral. A primeira é uma
prova como se segue:

Provar que a F(X)= Integral definida de x a 3x 1/t dt é constante no
intervalo (0,+infinito).

Se alguém puder, fornecer a habitual ajuda, agradeço muito. Marcelo.

---
iBestMail, agora com POP3/SMTP e 120MB de espaço!
Experimente: http://www.ibestmail.com.br
=
Instruções para entrar na lista, sair da lista e usar a lista em
http://www.mat.puc-rio.br/~nicolau/olimp/obm-l.html
=

=
Instruções para entrar na lista, sair da lista e usar a lista em
http://www.mat.puc-rio.br/~nicolau/olimp/obm-l.html
=


RE: [obm-l] Integral parte 2

2004-09-09 Por tôpico Leandro Lacorte Recova
F(x) = Int_{x,x+h} ln t dt 

Use integracao por partes.

U=ln(t) 
dV=dt 
du=1/t 
v=t 

F(x)=  lim 1/h[(uv-int(vdu))] em [x,x+h]
 h-0

F(x) = lim 1/h[t.ln(t)-int(t.1/t)] em [x,x+h] 
 h-0

F(x) = lim 1/h[(x+h).ln(x+h) - (x+h) - (x.ln(x) - x) ]
 h-0

F(x) = lim 1/h{x[ln(x+h)-ln(x)] + ln(x+h) - h]
 h-0 

F(x) = x lim (1/h){ln(x+h)-ln(x)]} + ln(x) -1 
   h-0 


O primeiro limite e a definicao da derivada de ln(x). Entao temos que 

F(x) = x.(1/x) + ln(x) - 1 = ln(x). 



Leandro. 
Los Angeles, CA. 

-Original Message-
From: [EMAIL PROTECTED] [mailto:[EMAIL PROTECTED] On
Behalf Of [EMAIL PROTECTED]
Sent: Thursday, September 09, 2004 2:09 PM
To: [EMAIL PROTECTED]
Subject: [obm-l] Integral parte 2

A 2ª questão é :

Ache o limite :

lim  Integral definida de x a x+h  Ln t dt / h
h-0

Valeu, Marcelo.



---
iBestMail, agora com POP3/SMTP e 120MB de espaço!
Experimente: http://www.ibestmail.com.br
=
Instruções para entrar na lista, sair da lista e usar a lista em
http://www.mat.puc-rio.br/~nicolau/olimp/obm-l.html
=

=
Instruções para entrar na lista, sair da lista e usar a lista em
http://www.mat.puc-rio.br/~nicolau/olimp/obm-l.html
=


RE: [obm-l] Integral (Provar que...) parte 1

2004-09-09 Por tôpico Leandro Lacorte Recova
Ao invest de 2x, eu queria colocar x. Foi mal !!! 

Entao fica ln(3) constante. 

Leandro. 

-Original Message-
From: [EMAIL PROTECTED] [mailto:[EMAIL PROTECTED] On
Behalf Of Leandro Lacorte Recova
Sent: Thursday, September 09, 2004 3:30 PM
To: [EMAIL PROTECTED]
Subject: RE: [obm-l] Integral (Provar que...) parte 1

Caso nao interpretei errado a questao, 

F(x)=ln(3x)-ln(2x)=ln(3x/2x)=ln(3/2) que e constante no intervalo de 0 a
+inf.

Leandro

-Original Message-
From: [EMAIL PROTECTED] [mailto:[EMAIL PROTECTED] On
Behalf Of [EMAIL PROTECTED]
Sent: Thursday, September 09, 2004 2:05 PM
To: [EMAIL PROTECTED]
Subject: [obm-l] Integral (Provar que...) parte 1

Olá pessoal da lista, boa noite.

Estou enviando três questões sobre a matéria integral. A primeira é uma
prova como se segue:

Provar que a F(X)= Integral definida de x a 3x 1/t dt é constante no
intervalo (0,+infinito).

Se alguém puder, fornecer a habitual ajuda, agradeço muito. Marcelo.

---
iBestMail, agora com POP3/SMTP e 120MB de espaço!
Experimente: http://www.ibestmail.com.br
=
Instruções para entrar na lista, sair da lista e usar a lista em
http://www.mat.puc-rio.br/~nicolau/olimp/obm-l.html
=

=
Instruções para entrar na lista, sair da lista e usar a lista em
http://www.mat.puc-rio.br/~nicolau/olimp/obm-l.html
=

=
Instruções para entrar na lista, sair da lista e usar a lista em
http://www.mat.puc-rio.br/~nicolau/olimp/obm-l.html
=


[obm-l] RE: [obm-l] Demonstração Teorema Laplace...

2004-08-19 Por tôpico Leandro Lacorte Recova








Num dos livros do Iezzi, a colecao de 10 livros,
tinha um apendice com a demonstracao. Acho que eles faziam por inducao. 



Fale com o Fabio Dias, pois ele mandou um
email pra lista algum tempo atras. Alias, a demonstracao estava muito bem apresentada
la tambem.



Leandro



-Original Message-
From: [EMAIL PROTECTED]
[mailto:[EMAIL PROTECTED] On Behalf
Of Alan Pellejero
Sent: Wednesday, August 18, 2004
7:23 PM
To: [EMAIL PROTECTED]
Subject: RE: [obm-l] Demonstração
Teorema Laplace...





Olá Leandro,





refiro-me àquele usado no ensino médio (abaixamento de ordem).





Procurei no google, mas não achei nada sobre.





Agradeço desde já!!!





ALAN

LEANDRO L RECOVA
[EMAIL PROTECTED] wrote:





Alan,

Existem varios teoremas associados a Laplace. Qual voce esta se referindo ?


From: Alan Pellejero 
<[EMAIL PROTECTED]>Reply-To: [EMAIL PROTECTED]
To: [EMAIL PROTECTED]
Subject: [obm-l] Demonstração Teorema Laplace...
Date: Wed, 18 Aug 2004 19:54:09 -0300 (ART)

Olá amigos da lista,
gostaria de saber onde eu posso encontrar a demonstração do teorema de 
Laplace.
Grato desde já pela ajuda,
Alan Pellejero


-
Yahoo! Acesso Grátis - navegue de graça com conexão de qualidade!


=
Instruções para entrar na lista, sair da lista e usar a lista em
http://www.mat.puc-rio.br/~nicolau/olimp/obm-l.html
=









Yahoo!
Acesso Grátis - navegue de graça com conexão de qualidade!








RE: [obm-l] essa foi no chute

2004-08-17 Por tôpico Leandro Lacorte Recova
Primeiro, vamos deduzir o que o problema quer:

a) Inverso Multiplicativo: Q/P.
b) Oposto do Inverso Multiplicativo: -Q/P

Portanto, o que o problema pede e encontrar x such that 

(P-X)/(Q-X) = -Q/P 

Isolando x, obtemos

X = ( p^2+q^2 ) / (p+q)

-Original Message-
From: [EMAIL PROTECTED] [mailto:[EMAIL PROTECTED] On
Behalf Of [EMAIL PROTECTED]
Sent: Tuesday, August 17, 2004 2:23 PM
To: [EMAIL PROTECTED]
Subject: Re: [obm-l] essa foi no chute 




Em 17 Aug 2004, [EMAIL PROTECTED] escreveu: 
Eu acho que deve ser isto 
(p-x)/(q-x)=-q/p(oposto do inverso multiplicativo) 
p^2-px=-q^2+qx 
x=(p^2+q^2)/(p+q) 
Alternativa C 
ALGEM PODERIA ME ESPLICAR ESTA QUESTAO! 
 (EPCAR)2005 VERSAO:C 
 26. SENDO P/Q UMA FRAÇAO IRREDUTIVEL, O NUMERO QUE 
DEVE SUBTRAIR DE SEUS TERMOS PARA SE OBTER O OPOSTO DO 
INVERSO MUTIPLICATIVO DESSA FRAÇAO É 
 A)P+Q C)((P^2)+(Q^2))/(P+Q) 
 B)-(P+Q) D)Q-P 
=== 
 O QUE SERIA O O OPOSTO DO INVERSO MUTIPLICATIVO 
 
__ 
Acabe com aquelas janelinhas que pulam na sua tela. 
AntiPop-up UOL - É grátis! 
http://antipopup.uol.com.br/ 
 
= 
Instruções para entrar na lista, sair da lista e usar a lista em 
http://www.mat.puc-rio.br/~nicolau/olimp/obm-l.html 
= 
 
-- 


_
Quer mais velocidade?
Só com o acesso Aditivado iG, a velocidade que você quer na hora que você
precisa.
Clique aqui: http://www.acessoaditivado.ig.com.br


=
Instruções para entrar na lista, sair da lista e usar a lista em
http://www.mat.puc-rio.br/~nicolau/olimp/obm-l.html
=


RE: [obm-l] fsrmula_de_transformagco_da_soma_em_produto II

2004-08-11 Por tôpico Leandro Lacorte Recova
Felipe,

Vou fazer so a primeira:

sin(a+b) = sin(a).cos(b) + cos(a).sin(b)
sin(a-b) = sin(a).cos(b) - cos(a).sin(b) 



Some as 2 equacoes, 

sin(a+b) + sin(a-b) = 2.sin(a).cos(b) 

Agora, faca a+b=x e a-b=y, voce encontrara que a=(x+y)/2 e b=(x-y)/2 

Logo,

Sin(x) + sin (y) = 2.sin((x+y)/2)cos((x-y)/2).

As outras saem de maneira analoga. 

Leandro
Los Angeles, CA.

-Original Message-
From: [EMAIL PROTECTED] [mailto:[EMAIL PROTECTED] On
Behalf Of Felipe Torres
Sent: Wednesday, August 11, 2004 10:16 AM
To: [EMAIL PROTECTED]
Subject: [obm-l] fsrmula_de_transformagco_da_soma_em_produto II

oi
eu n me referia a fórmula de adição de arcos, mas sim
às seguintes:

sen(x) + sen(y)= 2sen[(x+y)/2]*cos[(x-y)/2]

sen(x) - sen(y)= 2sen[(x-y)/2]*cos[(x+y)/2]

cos(x) + cos(y)= 2cos{(x+y)/2]*cos[(x-y)]/2]

cos(x) - cos(y)= -2sen[(x+y)/2]*sen[(x-y)/2]

de qualquer maneira, obrigado pela ajuda d antes, pois
eu n conhecia aquela dedução.

Felipe



__
Do you Yahoo!?
Yahoo! Mail - 50x more storage than other providers!
http://promotions.yahoo.com/new_mail
=
Instruções para entrar na lista, sair da lista e usar a lista em
http://www.mat.puc-rio.br/~nicolau/olimp/obm-l.html
=

=
Instruções para entrar na lista, sair da lista e usar a lista em
http://www.mat.puc-rio.br/~nicolau/olimp/obm-l.html
=


RE: [obm-l] questao simples do bartle

2004-08-11 Por tôpico Leandro Lacorte Recova
Gugu,

Eu dei uma olhada nos videos do IMPA para professores de ensino medio e
achei muito bom. O prof. Elon explica muito bem !! 

O que esta acontencendo com nosso Mengao  


Regards,

Leandro

-Original Message-
From: [EMAIL PROTECTED] [mailto:[EMAIL PROTECTED] On
Behalf Of Carlos Gustavo Tamm de Araujo Moreira
Sent: Wednesday, August 11, 2004 11:45 AM
To: [EMAIL PROTECTED]
Subject: Re: [obm-l] questao simples do bartle

   E' costume usar a notacao A^B para o conjunto de todas as funcoes de B em
A. Quando A e' um corpo isso e' um espaco vetorial sobre A.
   Abracos,
Gugu

   

Pessoal, este problema tirado do capitulo 8 (The Topology of Cartesian 
Spaces) me parece ser simples por ser um dos primeiros do capitulo. Eu 
realmente não entendi o enunciado. Me desculpem pelo ingles, se alguem 
quiser eu traduzo o enunciado.

Let S = {1,2,...,p}, for some p E N. Show that the vector space R^S
is essentially the same as the space R^p

Gostaria que alguem por favor me explicasse o que exatamente ele quer no 
problema ou seja, acredito que basta explicar como se mostra que um 
espaço vetorial é essencialmente o mesmo que um outro e tambem o que é 
R^S. S é um conjunto...soa estranho, estou acosumado com R^2, R^3 e de 
associar a ideia de produto cartesiano mas como imaginar para R^S onde S 
é um conjunto de numeros naturais?

obrigado
-- 
Niski - http://www.linux.ime.usp.br/~niski

[upon losing the use of his right eye]
Now I will have less distraction
Leonhard Euler

=
Instruções para entrar na lista, sair da lista e usar a lista em
http://www.mat.puc-rio.br/~nicolau/olimp/obm-l.html
=


=
Instruções para entrar na lista, sair da lista e usar a lista em
http://www.mat.puc-rio.br/~nicolau/olimp/obm-l.html
=

=
Instruções para entrar na lista, sair da lista e usar a lista em
http://www.mat.puc-rio.br/~nicolau/olimp/obm-l.html
=


RE: [obm-l] Limites

2004-07-27 Por tôpico Leandro Lacorte Recova
Paulo,

Na sua notacao do numero 1, quem esta elevado a lna/lnx ? E somente o
argumento de ln(x+1) ou o termo todo ln(x+1) ? 

Leandro.

-Original Message-
From: [EMAIL PROTECTED] [mailto:[EMAIL PROTECTED] On
Behalf Of paulobarclay
Sent: Tuesday, July 27, 2004 1:18 PM
To: obm-l
Subject: [obm-l] Limites

Pessoal, gostaria de uma ajuda.Estou com dificuldades 
em provar as seguintes afirmações.
 
1)prove que o lim ln((x+1)) ^(lna/lnx), qdo x tende a 
zero é igual a Lna.

2) Prove que Lim p(x)/e^x = 0 quando x tende a infinito.

Muito obrigado.

paulo barclay


 





 
__
Acabe com aquelas janelinhas que pulam na sua tela.
AntiPop-up UOL - É grátis!
http://antipopup.uol.com.br/



=
Instruções para entrar na lista, sair da lista e usar a lista em
http://www.mat.puc-rio.br/~nicolau/olimp/obm-l.html
=

=
Instruções para entrar na lista, sair da lista e usar a lista em
http://www.mat.puc-rio.br/~nicolau/olimp/obm-l.html
=


[obm-l] RE: [obm-l] Variedade Diferenciável

2004-07-23 Por tôpico Leandro Lacorte Recova









Esses termos voce encontra no livro do
Manfredo, geometria riemaniana. Logo no capitulo 0 voce encontrara a definicao
de variedade, variedade diferenciavel e variavel riemaniana vem logo no
seguinte capitulo. 



Manifold e uma variedade. Voce nao pode
traduzir Manifold como uma variedade diferenciavel.  



Smooth Manifold e uma variedade
diferenciavel. 



Sobre formas diferenciaveis, o Manfredo
tambem escreveu um livro sobre Formas Diferenciaveis que foi publicado pela
Springer. La voce encontra a definicao de forma diferenciavel sobre uma
variedade e ainda, a belissima generalizacao do teorema de Gauss-Bonnet para
Variedades Riemanianas feitas pelo Chern. 



Leandro





-Original Message-
From: [EMAIL PROTECTED]
[mailto:[EMAIL PROTECTED] On Behalf
Of Fernando Villar
Sent: Friday, July 23, 2004 1:26
PM
To: [EMAIL PROTECTED]
Subject: [obm-l] Variedade
Diferenciável





Olá amigos da lista!











Qual a definição de variedade?











O termo manifold pode ser traduzido
comovariedade diferenciável?











Smooth manifold pode ser traduzido como ?











Qual a definição de forma diferenciável sobre uma uma
variedade diferenciável?











Grato pela atenção!











Fernando
















RE: [obm-l] EDP

2004-07-07 Por tôpico Leandro Lacorte Recova








Cicero, nao falta uma segunda condicao de
contorno no problema 2 ? 



-Original Message-
From: [EMAIL PROTECTED]
[mailto:[EMAIL PROTECTED] On Behalf
Of [EMAIL PROTECTED]
Sent: Tuesday, July 06, 2004 7:49
PM
To: [EMAIL PROTECTED]
Subject: [obm-l] EDP











Tenho alguns 

exercícios de EDP que não consegui fazer, alguém poderia se
manifestar. 

 

1. Resolva o problema 

 

u_tt = u_xx + A em R, R = {(x,t) em R^2 / 0  x  L e t 
0} 

 

u(0,t) = 0, u(L,t) = e^(-t), t  0 

 

u(x,0) = u_t(x,0) = 0, 0  x  L, onde A é uma constante 

 

2. Estude o problema 

 

u_t = Ku_xx em R = {(x,t) em R^2 / 0  x  L e t  0} 

 

u(x,0) = f(x) para 0  x  L 

 

Obrigado 

 

Cícero Thiago 

 








=
Instruções para entrar na lista, sair da lista e usar a lista em
http://www.mat.puc-rio.br/~nicolau/olimp/obm-l.html
=

RE: [obm-l] Problemas com binomiais

2004-06-24 Por tôpico Leandro Lacorte Recova
Na segunda desigualdade, tente expandir o binomio (1+2)^n. Voce encontrara a
resposta imediato ! 

-Original Message-
From: [EMAIL PROTECTED] [mailto:[EMAIL PROTECTED] On
Behalf Of David M. Cardoso
Sent: Thursday, June 24, 2004 8:10 AM
To: [EMAIL PROTECTED]
Subject: [obm-l] Problemas com binomiais


Oi pessoal,

Gostaria da ajuda de voces com 2 questoes que não consigo fazer:
(qualquer comentario/ideia vai ajudar, eu não consigo sair do canto nessas
questoes)

http://www.suati.com.br/david/questao3.29.gif
http://www.suati.com.br/david/questao3.32.gif

[]'s
David


=
Instruções para entrar na lista, sair da lista e usar a lista em
http://www.mat.puc-rio.br/~nicolau/olimp/obm-l.html
=

=
Instruções para entrar na lista, sair da lista e usar a lista em
http://www.mat.puc-rio.br/~nicolau/olimp/obm-l.html
=


RE: [obm-l] polinomio interpolador na forma de newton

2004-06-16 Por tôpico Leandro Lacorte Recova
Tente o PRINCIPIA (Isaac Newton). 

Regards

Leandro
Los Angeles, CA

-Original Message-
From: [EMAIL PROTECTED] [mailto:[EMAIL PROTECTED] On
Behalf Of niski
Sent: Wednesday, June 16, 2004 11:56 AM
To: [EMAIL PROTECTED]
Subject: [obm-l] polinomio interpolador na forma de newton

Estou estudando interpolacao polinomial pelo livro da Ana Flora Humes, 
Ines Homem de Melo, Luzia Yoshida e Wagner Tunis Martins. O livro é 
muito bom, mas particularmente nessa parte do polinomio interpolador na 
forma de newton as provas sao na maior parte feitas por indução 
sonolentas e gigantes. Lamentavel o livro nao traga referencias 
historicas, mas acredito que o metodo foi criado por Newton e certamente 
ele nao usou inducao para chegar aos mesmos resultados. Assim eu 
pergunto: Alguem conhece algum lugar (site, livro) onde eu possa ver as 
ideias originais do Newton?
Obrigado.

-- 
Niski - http://www.linux.ime.usp.br/~niski

[upon losing the use of his right eye]
Now I will have less distraction
Leonhard Euler

=
Instruções para entrar na lista, sair da lista e usar a lista em
http://www.mat.puc-rio.br/~nicolau/olimp/obm-l.html
=

=
Instruções para entrar na lista, sair da lista e usar a lista em
http://www.mat.puc-rio.br/~nicolau/olimp/obm-l.html
=


RE: [obm-l] polinomio interpolador na forma de newton

2004-06-16 Por tôpico Leandro Lacorte Recova
Eu estava brincando. A ideia do Morgado e excelente.  

Leandro. 

-Original Message-
From: [EMAIL PROTECTED] [mailto:[EMAIL PROTECTED] On
Behalf Of niski
Sent: Wednesday, June 16, 2004 1:14 PM
To: [EMAIL PROTECTED]
Subject: Re: [obm-l] polinomio interpolador na forma de newton

Poxa ai voce exagerou. Quero as ideias dele mas nas notacoes e 
vocabulario atual. Fora que eu nem sei se ele trata disso no Principia.
Vou seguir a ideia do Morgado.

Leandro Lacorte Recova wrote:

 Tente o PRINCIPIA (Isaac Newton). 
 
 Regards
 
 Leandro
 Los Angeles, CA
 
 -Original Message-
 From: [EMAIL PROTECTED] [mailto:[EMAIL PROTECTED] On
 Behalf Of niski
 Sent: Wednesday, June 16, 2004 11:56 AM
 To: [EMAIL PROTECTED]
 Subject: [obm-l] polinomio interpolador na forma de newton
 
 Estou estudando interpolacao polinomial pelo livro da Ana Flora Humes, 
 Ines Homem de Melo, Luzia Yoshida e Wagner Tunis Martins. O livro é 
 muito bom, mas particularmente nessa parte do polinomio interpolador na 
 forma de newton as provas sao na maior parte feitas por indução 
 sonolentas e gigantes. Lamentavel o livro nao traga referencias 
 historicas, mas acredito que o metodo foi criado por Newton e certamente 
 ele nao usou inducao para chegar aos mesmos resultados. Assim eu 
 pergunto: Alguem conhece algum lugar (site, livro) onde eu possa ver as 
 ideias originais do Newton?
 Obrigado.
 

-- 
Niski - http://www.linux.ime.usp.br/~niski

[upon losing the use of his right eye]
Now I will have less distraction
Leonhard Euler

=
Instruções para entrar na lista, sair da lista e usar a lista em
http://www.mat.puc-rio.br/~nicolau/olimp/obm-l.html
=

=
Instruções para entrar na lista, sair da lista e usar a lista em
http://www.mat.puc-rio.br/~nicolau/olimp/obm-l.html
=


[obm-l] RE: [obm-l] livro de teoria da medida e integração

2004-06-15 Por tôpico Leandro Lacorte Recova
O livro do Royden tambem e bom. 

-Original Message-
From: [EMAIL PROTECTED] [mailto:[EMAIL PROTECTED] On
Behalf Of Gustavo Salgueiro
Sent: Tuesday, June 15, 2004 9:59 AM
To: [EMAIL PROTECTED]
Subject: [obm-l] livro de teoria da medida e integração

Gostaria de saber qual livro sobre medida e integração as pessoas do grupo 
acham o melhor. Geralmente estes livros são importados e caros, daí acho 
importante saber direito qual comprar. Já me falaram de um do bartle(theory 
of integration) e um do rudin(real and complex analysis);qual desses é 
melhor? (e tem outros melhores?)

Obrigado

_
MSN Hotmail, o maior webmail do Brasil.  http://www.hotmail.com

=
Instruções para entrar na lista, sair da lista e usar a lista em
http://www.mat.puc-rio.br/~nicolau/olimp/obm-l.html
=

=
Instruções para entrar na lista, sair da lista e usar a lista em
http://www.mat.puc-rio.br/~nicolau/olimp/obm-l.html
=


[obm-l] RE: [obm-l] livro de teoria da medida e integração

2004-06-15 Por tôpico Leandro Lacorte Recova








Try this link my friend:



http://www.amazon.com/exec/obidos/tg/detail/-/0853123373/qid=1087333272/sr=1-1/ref=sr_1_1/102-3249771-8883364?v=glances=books




Leandro. 



-Original Message-
From: [EMAIL PROTECTED]
[mailto:[EMAIL PROTECTED] On Behalf
Of neylor farias magalhaes
Sent: Tuesday, June 15, 2004 1:29
PM
To: [EMAIL PROTECTED]
Subject: Re: [obm-l] livro de
teoria da medida e integração





G.D. Barra é o autor e Springer a
editora?





vc podia por o titulo ou o isbn para facilitar a busca?:)

Danilo notes
[EMAIL PROTECTED] wrote:








O livro do G.D. Barra da Springer é muito bom, tem dezenas de
exercicios resolvidos.

















 Abs.
Gustavo Salgueiro
[EMAIL PROTECTED] wrote:





Gostaria de saber qual livro sobre medida e integração as pessoas do
grupo 
acham o melhor. Geralmente estes livros são importados e caros, daí acho 
importante saber direito qual comprar. Já me falaram de um do bartle(theory 
of integration) e um do rudin(real and complex analysis);qual desses é 
melhor? (e tem outros melhores?)

Obrigado

_
MSN Hotmail, o maior webmail do Brasil. http://www.hotmail.com

=
Instruções para entrar na lista, sair da lista e usar a lista em
http://www.mat.puc-rio.br/~nicolau/olimp/obm-l.html
=











Yahoo! Mail - Participe da pesquisa
global sobre o Yahoo! Mail. Clique
aqui!













Yahoo! Messenger - Fale com seus amigos
online. Instale
agora!








RE: [obm-l] integral de tg(x)

2004-06-11 Por tôpico Leandro Lacorte Recova
Caro Andre,

Faca assim: 

tg(x) = sin(x)/cos(x) 

Faca, u = cos(x), entao du=-sin(x)dx 

Assim,  a integral fica


Int[tg(x)dx] = Int[sin(x)/cos(x)]dx = Int[-du/u] = 

= -ln(u) + C = ln(1/u) + C = ln(1/cos(x)) + C = ln(sec(x)) + C. 


Regards,

Leandro. 



-Original Message-
From: [EMAIL PROTECTED] [mailto:[EMAIL PROTECTED] On
Behalf Of André Martin Timpanaro
Sent: Friday, June 11, 2004 11:51 AM
To: [EMAIL PROTECTED]
Subject: [obm-l] integral de tg(x)

Estou tendo problemas para encontrar a primitiva de tg(x), se alguém puder 
me ajudar agradeço.

André T.

_
MSN Messenger: converse com os seus amigos online.  
http://messenger.msn.com.br

=
Instruções para entrar na lista, sair da lista e usar a lista em
http://www.mat.puc-rio.br/~nicolau/olimp/obm-l.html
=

=
Instruções para entrar na lista, sair da lista e usar a lista em
http://www.mat.puc-rio.br/~nicolau/olimp/obm-l.html
=


[obm-l] RE: [obm-l] O Último Teorema de Fermat

2004-06-10 Por tôpico Leandro Lacorte Recova
Eu li esse livro ha alguns anos atras:

http://www.amazon.com/exec/obidos/tg/detail/-/1857026691/qid=1086894781/sr=1
-10/ref=sr_1_10/102-3249771-8883364?v=glances=books 

E te da uma boa ideia da demonstracao. 

Leandro

-Original Message-
From: [EMAIL PROTECTED] [mailto:[EMAIL PROTECTED] On
Behalf Of Nicolau C. Saldanha
Sent: Thursday, June 10, 2004 11:48 AM
To: [EMAIL PROTECTED]
Subject: Re: [obm-l] O Último Teorema de Fermat

On Wed, Jun 09, 2004 at 02:18:17PM -0300, Henrique Patrício Sant'Anna Branco
wrote:
 Pessoal,
 
 Tenho ouvido muita coisa sobre esse teorema na faculdade e gostaria de
 saber, de vocês, se o caso geral já foi demonstrado. Sei que o próprio
 Fermat provou sua validade quando 4|n.
 
 Pra quem não sabe do que estou falando, aí vai o enunciado:
 
 A equação diofantina x^n + y^n = z^n não é solúvel por nenhum triplo (x,
y,
 z), com x, y, z E N, se n  2.

Supondo que para você N signifique {1,2,3,...} então sim, isto é um teorema.

A comunidade matemática tem uma demonstração realmente muito engenhosa
deste resultado mas ela não caberia na margem de tamanho das mensagens
permitidas nesta lista (2 caracteres).

[]s, N.

PS: Desculpem, não resisti.
=
Instruções para entrar na lista, sair da lista e usar a lista em
http://www.mat.puc-rio.br/~nicolau/olimp/obm-l.html
=

=
Instruções para entrar na lista, sair da lista e usar a lista em
http://www.mat.puc-rio.br/~nicolau/olimp/obm-l.html
=


[obm-l] RE: [obm-l] Fórmula para ordenar números primos

2004-06-10 Por tôpico Leandro Lacorte Recova
Nicolau,

Tem como fazer o download desse livro sem ter que ir pagina por pagina ? 

Saudacoes,

Leandro

-Original Message-
From: [EMAIL PROTECTED] [mailto:[EMAIL PROTECTED] On
Behalf Of Nicolau C. Saldanha
Sent: Thursday, June 10, 2004 11:58 AM
To: [EMAIL PROTECTED]
Subject: Re: [obm-l] Fórmula para ordenar números primos

On Thu, Jun 10, 2004 at 11:36:03AM -0400, [EMAIL PROTECTED] wrote:
 Dada a fórmula X. Através dela, podemos ordenar todos os números primos
numa
 sequencia lógica e completamente controlada. Podemos prever e dizer com
 exatidão, onde se encontram os primos e quantos números naturais pulamos
 para encontrá-los. Essa dada fórmula tem alguma importância ??? 

Eu sugiro que você leia a primeira seção do capítulo 3 do meu livro com
Gugu sobre primos de Mersenne. Está na minha home page e pode ser comprado
pelo Impa.

http://www.mat.puc-rio.br/~nicolau/publ/papers/mersenne

[]s, N.
=
Instruções para entrar na lista, sair da lista e usar a lista em
http://www.mat.puc-rio.br/~nicolau/olimp/obm-l.html
=

=
Instruções para entrar na lista, sair da lista e usar a lista em
http://www.mat.puc-rio.br/~nicolau/olimp/obm-l.html
=


RE: [obm-l] determinantes

2004-05-25 Por tôpico Leandro Lacorte Recova








Fael,



Uma coisa que eu fui
aprender depois de muito tempo e que a gente tem que dar um tempo as coisas. EU
tambem tinha vontade de saber de imediato a aplicacao de diversas coisas do 2º grau,
como por exemplo determinantes e complexos no seu caso, mas existe todo um
fluxo de aprendizado pelo qual voce tem que passar pra ficar pronto para
entender as aplicacoes. Numeros Complexos e Algebra Linear sao materias
incriveis e de grande poder. Sei que as vezes a gente fica aprendendo varias
coisas sem ver onde vai aplicar isso, mas as aplicacoes demandam conhecimentos
de varias areas. Exemplos:  



Numeros Complexos: Toda
teoria eletromagnetica gira em torno de algebra de numeros complexos. Teoria de
Processamento Digital de Sinais esta toda em cima disso tambem (Transformada Z,
Transformadas de Wavelet, etc). 



Algebra Linear:
Determinantes sao importantes pra voce entender algumas solucoes de sistemas de
equacoes diferenciais que aparecem em sistemas de controle e robotica, determinacao
de autovalores para analise de estabilidade esta baseada em voce calcular um
determinante especial, etc. 



Espere mais um pouco que
elas virao em cheio ! Tive um professor que falava que se o mundo acabasse
hoje, bastaria ter somente dois livros pra reconstrui-lo: Uma biblia e um de
Transformadas de Fourier.  



Nao vou me estender pra
ficar off-topic. 



Leandro

Los Angeles, CA. 



-Original Message-
From: [EMAIL PROTECTED]
[mailto:[EMAIL PROTECTED] On Behalf
Of [EMAIL PROTECTED]
Sent: Monday, May 24, 2004 8:47 PM
To: [EMAIL PROTECTED]
Subject: Re: [obm-l] determinantes



Pegando um gancho: 

De todos os conceitos matematicos de Ensino Medio, os unicos que ate agora eu
nao vejo contextualizacao sao *determinantes* e *numeros complexos*. Sei que
ambos estao presentes na historia da criacao dos computadores, por exemplo, mas
nao consigo imaginar uma situacao-problema em que seja necessario utilizar
estes 2 conceitos. Todos os outros conceitos de matematica de Ensino Medio sao
facilmente contextualizados, mas estes 2 sao um *estranho no ninho* da
matematica de Ensino Medio. E para piorar, muitos livros definem *determinante*
como um numero associado a uma matriz (Grande definicao ! Ironicamente falando
:-) 



Em uma mensagem de 25/5/2004 00:29:48 Hora padrão leste da Am. Sul,
[EMAIL PROTECTED] escreveu: 







olá, gostaria de saber se existe uma definição exata de determinante de uma
matriz... 

é que eu já vi 3 definições distintas e gostaria de saber se todas sao aceitas
como definições mesmo, ou apenas uma delas é a certa e as outras sao teoremas a
partir dessa, ou é ainda uma outra além dessa 3... 

uma das definições, dada pelo Manoel Paiva, vol 2 é: 
O determinante de uma matriz quadrada A = (a_ij)_(nXn), com n = 2, é
igual ao produto dos elementos da diagonal principal de qualquer matriz
triangular B, equiparável a A. 

bom, nesse caso eu gostaria de saber se existe algum lugar em que eu posso
encontra a demonstração desses dois teoremas: 

Dada uma matriz quadrada A = (a_ij)_(nXn), existe uma matriz triangular B
= (b_ij)_(nXn) equiparável a A. 
esse eu acho meio intuitivo, mas tentei provar matematicamente e não
consegui... 


Se duas matrizes triangulares A e B são equiparáveis, então ambas possuem
o mesmo produto dos elementos da diagonal principal. 
esse nao é nem um pouco intuitivo e tb nao consegui demonstrar. 

bom, a outra definição que encontrei para determinante foi no Gelson Iezzi vol.
4.: 
O determinante de uma matriz de ordem n = 2 é a soma dos produtos dos
elementos da primeira coluna pelos respectivos cofatores. 

a outra definição que encontrei foi em um e-mail enviado para esta lista, por
Hugo Iver Vasconcelos Gonçalves: 
o determinante de uma matriz é a soma algébrica de todos os possíveis
fatores em que estão presentes um (e apenas um) elemento de cada linha e cada
coluna, sendo que aqueles em que os índices dos elementos da matriz
formam uma permutação de primeira classe são tomados positivamente e os demais,
negativamente. 
nesse caso a explicação que ele deu para permutação de primeira classe foi: 
permutação de primeira classe é aquela em que o número de inversões é
par 
e a explicação para inversões foi: 
inversão é o fato de um par de elementos de uma permutação não aparecer
na mesma ordem que apareceram na permutação inicial. No caso de a
permutação inicial de n números ser a disposição deste em ordem crescente, uma
inversão seria basicamente o fato de aparecer um número maior antes de um
menor. E se a ordem inicial deles for outra, pode-se sempre chamar o 1o
elemento de a1 e o n-ésimo de an, de modo que uma inversão será simplesmente
quando aparecer um número ap antes de um aq, tais que p  q. 

nesse caso eu nao entendi como calcular quantas inversoes foram necessarias
para chegar a dada permutação... 


bom, é isso, sanadas minha dúvidas e se não for abuso, gostaria de saber onde
poderia encontrar a demonstração do teorema fundamental de Laplace. 

desde já 

RE: Re[2]: [obm-l] Livro de eq Diferenciais...

2004-05-18 Por tôpico Leandro Lacorte Recova
O livro e caracterizado por apresentar diversos exemplos praticos em
cinematica, circuitos lineares, calculo numerico, etc. Na biblioteca da sua
universidade devem ter varios exemplares.

Leandro

-Original Message-
From: [EMAIL PROTECTED] [mailto:[EMAIL PROTECTED] On
Behalf Of Igor GomeZZ
Sent: Tuesday, May 18, 2004 12:29 PM
To: Augusto Cesar de Oliveira Morgado
Subject: Re[2]: [obm-l] Livro de eq Diferenciais...


Em 18/5/2004, 06:32, Augusto ([EMAIL PROTECTED]) disse:

 Boyce - Di Prima

Algum motivo em especial Professor? Existem exemplos/exercícios de cenas
reais?

Ateh mais!

   Igor GomeZZ   
 MirandaIM: ICQ# 29249895 / MSN [EMAIL PROTECTED]
 Vitória, Espírito Santo, Brasil
 Criação: 18/5/2004 (16:24)
#
Pare para pensar:

Ou nós encontramos um caminho, ou abrimos um. (Aníbal)   

#

=
Instruções para entrar na lista, sair da lista e usar a lista em
http://www.mat.puc-rio.br/~nicolau/olimp/obm-l.html
=

=
Instruções para entrar na lista, sair da lista e usar a lista em
http://www.mat.puc-rio.br/~nicolau/olimp/obm-l.html
=


RE: [obm-l] Livro de eq Diferenciais...

2004-05-18 Por tôpico Leandro Lacorte Recova
Eu ja lecionei esse curso e usei o Boyce e o do Tanaembaum: 

http://www.amazon.com/exec/obidos/tg/detail/-/0486649407/qid=1084917816/sr=1
-2/ref=sr_1_2/002-2320800-7444838?v=glances=books

O livro tem diversos exercicios e seria mais como referencia pra exercicios
e aprofundamento. O livro se chama Ordinary Differential Equations. 

Mas o Boyce e a referencia de quase todas as universidades no Brasil. E ele
nao custa 400 reais mesmo comprando na amazon. Ele custa 110 dolares se voce
comprar por aqui. Agora, siga a diga do Daniel e compre no Saraiva ao inves
de obter uma copia ilegal. 

http://www.amazon.com/exec/obidos/tg/detail/-/0471319996/qid=1084917816/sr=1
-8/ref=sr_1_8/002-2320800-7444838?v=glances=books 

Leandro. 

-Original Message-
From: [EMAIL PROTECTED] [mailto:[EMAIL PROTECTED] On
Behalf Of Daniel Silva Braz
Sent: Tuesday, May 18, 2004 2:38 PM
To: [EMAIL PROTECTED]
Subject: Re: [obm-l] Livro de eq Diferenciais...

 --- niski [EMAIL PROTECTED] escreveu:  No IME (SP) o
livro na ementa é o do Brauer e Nohel.
 O livro do Boyce esta disponivel nas redes de p2p

Bem...só lembrando..isso é ilegal !!! Se vc não
liga..então vá em frente..o risco é seu...

 para quem nao quiser 
 desenbolsar 400 reais.

não são 400 reais..são 106..continua sendo
muito..mas..

http://www.livrariasaraiva.com.br/produto/produto.dll/detalhe?pro_id=354135;
ID=C89B1B027D4050D1003080882

 
 Igor GomeZZ wrote:
 
  Em 18/5/2004, 06:32, Augusto
 ([EMAIL PROTECTED]) disse:
  
  
 Boyce - Di Prima
  
  
  Algum motivo em especial Professor? Existem
 exemplos/exercícios de cenas
  reais?
  
  Ateh mais!
  
     Igor GomeZZ  
 
   MirandaIM: ICQ# 29249895 / MSN
 [EMAIL PROTECTED]
   Vitória, Espírito Santo, Brasil
   Criação: 18/5/2004 (16:24)
 

#
  Pare para pensar:
  
  Ou nós encontramos um caminho, ou abrimos um.
 (Aníbal)   
  
 

#
  
 

=
  Instruções para entrar na lista, sair da lista e
 usar a lista em
 
 http://www.mat.puc-rio.br/~nicolau/olimp/obm-l.html
 

=
  
  
 
 -- 
 Niski - http://www.linux.ime.usp.br/~niski
 
 [upon losing the use of his right eye]
 Now I will have less distraction
 Leonhard Euler
 

=
 Instruções para entrar na lista, sair da lista e
 usar a lista em
 http://www.mat.puc-rio.br/~nicolau/olimp/obm-l.html

= 

__

Yahoo! Messenger - Fale com seus amigos online. Instale agora! 
http://br.download.yahoo.com/messenger/
=
Instruções para entrar na lista, sair da lista e usar a lista em
http://www.mat.puc-rio.br/~nicolau/olimp/obm-l.html
=

=
Instruções para entrar na lista, sair da lista e usar a lista em
http://www.mat.puc-rio.br/~nicolau/olimp/obm-l.html
=


RE: [obm-l] + duvidas

2004-05-14 Por tôpico Leandro Lacorte Recova
Voce pode montar o seguinte sistema:

Seja f(x)=ax^2 + bx + c = a(x-x1)(x-x2)

9a - 3b + c = 0 (1)
a + b + c = 0  (2) 

Dessas duas equacoes, isolando c, chegamos a igualdade  2a = b.

Sabemos que a abscissa do vertice da parabola e dada por: 

Xv = -b/2a = Xv = - 1 

Portanto, temos a 3a equacao procurada:

8 = a - b + c (3)

Precisamos somente encontrar o valor de a para o problema. 

Agora e contigo  

Regards,

Leandro. 



-Original Message-
From: [EMAIL PROTECTED] [mailto:[EMAIL PROTECTED] On
Behalf Of aryqueirozq
Sent: Friday, May 14, 2004 11:37 AM
To: [EMAIL PROTECTED]
Subject: [obm-l] + duvidas



A função f(x) do segundo grau tem raízes -3 e 1. A 
ordenada do vértice da parábola, gráfico de f(x), é 
igual a 8.
A única afirmativa VERDADEIRA sobre f(x) é
a) f(x) = -2(x-1)(x+3)
b) f(x) = -(x-1)(x+3)
c) f(x) = -2(x+1)(x-3)
d) f(x) = (x-1)(x+3)
e) f(x) = 2(x+1)(x-3)

Agradeço desde de já.
 
__
Acabe com aquelas janelinhas que pulam na sua tela.
AntiPop-up UOL - É grátis!
http://antipopup.uol.com.br/



=
Instruções para entrar na lista, sair da lista e usar a lista em
http://www.mat.puc-rio.br/~nicolau/olimp/obm-l.html
=

=
Instruções para entrar na lista, sair da lista e usar a lista em
http://www.mat.puc-rio.br/~nicolau/olimp/obm-l.html
=


RE: [obm-l] Demonstracoes no ensino medio

2004-05-10 Por tôpico Leandro Lacorte Recova








Na UnB, essa material e
obrigatoria tanto no Bacharelado como na Licenciatura. 



-Original Message-
From: [EMAIL PROTECTED]
[mailto:[EMAIL PROTECTED] On Behalf
Of Johann Peter Gustav Lejeune Dirichlet
Sent: Monday, May 10, 2004 12:25
PM
To: [EMAIL PROTECTED]
Subject: Re: [obm-l] Demonstracoes
no ensino medio





Como quiser chamar, Teoria dos Numeros. Essa materia e opcional aqui na
USP Sao Carlos para o curso de Matematica.E ate divertioda, mas esperar a
faculdade para fazer a OBM nivel 3 ja mostra como a coisa ta andando...

niski
[EMAIL PROTECTED] wrote: 

Aritimética dos Inteiros... Quase ninguem da minha geracao tem 
conhecimentos basicos do assunto. O motivo, acredito que é das duas uma; 
Ou isso foi ensinado em uma epoca errada (i.e o cerebro do aluno, em 
media, nessa idade, nao esta preparado para tal refinamento de ideias) 
ou isso foi ensinado no modo bitolacao. Eu só fui estudar algoritmo de 
Euclides no cursinho.


 Eu sou professor na PUC, e garanto a você que a quase totalidade dos
alunos,
 mesmo os melhores, tem muito a aprender com um curso destes. Para tomar
 um exemplo bem concreto: o conceito de mdc e as formas de calculá-lo
 são matéria de primário mas acho que nem um por cento dos calouros saberia
 descrever o algoritmo de Euclides com demonstração. 
 


-- 
Niski - http://www.linux.ime.usp.br/~niski

[upon losing the use of his ! right eye]
Now I will have less distraction
Leonhard Euler

=
Instruções para entrar na lista, sair da lista e usar a lista em
http://www.mat.puc-rio.br/~nicolau/olimp/obm-l.html
=r/~nicolau/olimp/obm-l.html
=









TRANSIRE SVVM PECTVS MVNDOQVE POTIRI 

CONGREGATI EX TOTO ORBE MATHEMATICI OB SCRIPTA INSIGNIA TRIBVERE 

Fields Medal(John Charles Fields)











N.F.C. (Ne Fronti Crede)













Yahoo! Messenger - Fale com seus amigos
online. Instale
agora!








RE: [obm-l] Demonstracoes no ensino medio

2004-05-10 Por tôpico Leandro Lacorte Recova








Comercei ??? 



-Original Message-
From: [EMAIL PROTECTED]
[mailto:[EMAIL PROTECTED] On Behalf
Of Johann Peter Gustav Lejeune Dirichlet
Sent: Monday, May 10, 2004 12:19
PM
To: [EMAIL PROTECTED]
Subject: Re: [obm-l] Demonstracoes
no ensino medio





Pra falar a verdade eu so aprendi demonstraçoes quando eu comercei a
fazer olimpiadas.

Nicolau C.
Saldanha [EMAIL PROTECTED] wrote: 

On Sun, May 09, 2004 at 11:01:26PM -0300, Johann Peter Gustav Lejeune
Dirichlet
wrote:
 So uma coisa que talvez seja util voces saberem: na faculdade a turma de
 matematica aqui da USP-Sao Carlos tem aula de MEB (Matematica do Ensino
 Basico). Curioso, eu perguntei o que sec aprende nessa matera e a resposta
 foi um belo de um  Tudo o que se ve no Ensino Medio, com
demonstraçoes! .
 Isto nao e estupido?

Eu sou professor na PUC, e garanto a você que a quase totalidade dos alunos,
mesmo os melhores, tem muito a aprender com um curso destes. Para tomar
um exemplo bem concreto: o conceito de mdc e as formas de calculá-lo
são matéria de primário mas acho que nem um por cento dos calouros saberia
descrever o algoritmo de Euclides com demonstração. 

[]s, N.
=
Instruções para entrar na lista, sair da lista e usar a lista em
http://www.mat.puc-rio.br/~nicolau/olimp/obm-l.html
=r/~nicolau/olimp/obm-l.html
=









TRANSIRE SVVM PECTVS MVNDOQVE POTIRI 

CONGREGATI EX TOTO ORBE MATHEMATICI OB SCRIPTA INSIGNIA TRIBVERE 

Fields Medal(John Charles Fields)











N.F.C. (Ne Fronti Crede)













Yahoo! Messenger - Fale com seus amigos
online. Instale
agora!








[obm-l] RE: [obm-l] Re: [obm-l]_Feiticeira_de_Gauss,_Geometria_hiperbólica_e_Riemanianna

2004-04-27 Por tôpico Leandro Lacorte Recova








Alan, 



Sobre a sua primeira
pergunta, nao sei de onde voce tirou isso ! 



Se quiser ler sobre
geometria riemaniana va ao www.google.com e
procure algo por la. 



Ou se quiser ler algo e
ja tem um background de geometria diferencial classica , calculo avancado,
algebra linear, pode pegar o livro do Manfredo Geometria Riemaniana
e vera o que isso significa. 





Leandro. 



-Original Message-
From: [EMAIL PROTECTED]
[mailto:[EMAIL PROTECTED] On Behalf
Of Alan Pellejero
Sent: Monday, April 26, 2004 1:42
PM
To: [EMAIL PROTECTED]
Subject: Re: [obm-l] Re:
[obm-l]_Feiticeira_de_Gauss,_Geometria_hiperbólica_e_Riemanianna






Por acaso a riemanianna é projetada numa circunferência??





E, na teoria da relatividade, que eu já andei olhando alguma coisa, por
que a massa varia???





Obrigado





Alan Pellejero
Nicolau C.
Saldanha [EMAIL PROTECTED] wrote:





On Mon, Apr 26, 2004 at 12:53:00PM -0300, Alan Pellejero wrote:
 Pessoal, desculpem-me se este assunto for off-topic, na verdade eu nem sei
o
 que não é off-topic, mas eu gostaria de saber o que é a feiticeira de
gauss.

Acho que a reclamação do Claudio não foi no sentido de que a sua mensagem
fosse off-topic, foi no sentido de dizer que a pergunta já tinha sido
respondida.

As perguntas abaixo não são off-topic mas exigem livros inteiros
para responder. Vou dar respostas sumárias.

 Gostaria de saber também o seguinte:
 
 se a geometria euclidiana é projetada no plano, onde é projetada a
 riemanianna e a hiperbólica???

Esta pergunta não faz sentido.

 Ouvi tambeém comentários sobre uma outra geometria que fazia a projeção
num
 plano que parece uma cela de cavalo...

O plano hipe! rbólico tem curvatura negativa; a sela de cavalo também tem;
as semelhanças param mais ou menos por aí.

 Por que tantas geomeetrias???

Por que são úteis. Existem tantas geometrias quanto os mátemáticos são
capazes de inventar e as mais úteis ficam famosas.

 Existem outras além dessas?

Muitas.

 Por que Einstein precisou da riemanianna pra formular a teoria da
 relatividade???

Acho que para responder esta pergunta você precisa primeiro estudar
os dois assuntos, não? Existem um monte de livros bons.

[]s, N.
=
Instruções para entrar na lista, sair da lista e usar a lista em
http://www.mat.puc-rio.br/~nicolau/olimp/obm-l.html
=r/~nicolau/olimp/obm-l.html
=











Yahoo! Messenger - Fale com seus amigos
online. Instale
agora!








FW: [obm-l] Gradiente

2004-03-24 Por tôpico Leandro Lacorte Recova
Esse conceito tambem e muito usado quando voce estuda Eletromagnetismo e
Teoria de Antenas e Teoria de Fluidos Classica. Nao vou me prolongar, pois
sera off-topic. A explicacao do Arthur esta otima. 

Leandro. 

-Original Message-
From: [EMAIL PROTECTED] [mailto:[EMAIL PROTECTED] On
Behalf Of Artur Costa Steiner
Sent: Wednesday, March 24, 2004 1:16 PM
To: [EMAIL PROTECTED]
Subject: Re: [obm-l] Gradiente

Uma referencia on line estah no Math World,
http://mathworld.wolfram.com/Gradient.html

O Math World naum eh de forma alguma uma fonte completa, mas dah uma
indicacao sobre o conceito procurado. 

O gradiente d uma funcao real f(x1..., x_n) eh um vetor do R^n cujas
componentes sao as derivadas parciais de f com relacao a cada uma das
variaveis x1...x_n (asumindo-se que tais derivadas existam no ponto em
questao). O gradiente eh um conceito muito util eh muito usado em problemas
de programacao matematica, quando se deseja determinar o minimo ou o maximo
de uma funcao de diversas variaveis. Pode-se demonstrar que, se vc estah em
um ponto (x1x_n), entao o maximo aumento de f eh obtido quando vc
desliza na direcao de seu gradiente (eh a maxima reducao eh obtida na
diracao oposta ao gradiente). Esta conclusao eh muito utilizada por
algoritmos de otimizacao. 
Se f passa por um maximo ou um minimo em um ponto interior de seu dominio e
seu gradinete existe neste ponto, entao ele eh nulo, quer dizer, todas as
derivadas parciais se anulam. 
 
O assunto eh extenso, para entende-lo vc deve consultar um bom livro de
Calculo ou de Analise.
Artur 


- Mensagem Original 
De: [EMAIL PROTECTED]
Para: [EMAIL PROTECTED] [EMAIL PROTECTED]
Assunto: [obm-l] Gradiente
Data: 23/03/04 22:26


Olá Pessoal

Vcs sabem onde eu posso encontrar material on-line sobre gradiente de fácil
de entender, com uma visão prática sobre o assunto.

Se vcs puderem explicar sobre gradientes para mim eu agradeço.

Pérsio 




Yahoo! Mail - O melhor e-mail do Brasil. Abra sua conta agora! 


OPEN Internet
@ Primeiro provedor do DF com anti-vírus no servidor de e-mails @


=
Instruções para entrar na lista, sair da lista e usar a lista em
http://www.mat.puc-rio.br/~nicolau/olimp/obm-l.html
=

=
Instruções para entrar na lista, sair da lista e usar a lista em
http://www.mat.puc-rio.br/~nicolau/olimp/obm-l.html
=


[obm-l] RE: Desculpe-me pela insistência Re: [obm-l] Integral - Cardióde

2004-03-23 Por tôpico Leandro Lacorte Recova








Ha um documento
interessante sobre a cardioide em:



http://www.mathcurve.com/courbes2d/cardioid/cardioid.shtml




Se voce le frances, nao
ha problema ! Ha um Gif
animado interessante mostrando que ela tambem e uma epicicloide. De uma olhada.



Leandro

Los Angeles, CA. 



-Original Message-
From: [EMAIL PROTECTED]
[mailto:[EMAIL PROTECTED] On Behalf
Of Roney Kevin
Sent: Tuesday, March
 23, 2004 7:35 AM
To: [EMAIL PROTECTED]
Subject: Desculpe-me pela
insistência Re: [obm-l] Integral - Cardióde





Agradeço a Johann Peter por ter tentado me ajudar.





No entanto, mesmo assim não consegui fazer a questão





.





 achar o volume do corpo formado pela rotação da cardióder
= a(1+cos teta) em torno do eixo polar.





...






Estou pagando Calculo I, to terminando o semstre agora. Sendo que o professor
passou uma lista de questões como a mostrada acima. O problema é que eu fui
pedir ajuda a ele e ele mesmo se enrolou não acertou fazer. Vasculhei na
internet pra ver se conseguia encontrar algo que me ajudasse, porém o melhor q
consegui foi um site q me fornecia uma fórmula para calcular o volume
envolvendo coordenadas polares, ate ai otimo se nao fosse o detalhe de q
a formúla usa integral dupla e ate agora em Calculo I so vemos ate a
integral unica.





Sei q o objetivo da lista não e pra tratar de problemas como o meu, no
entanto apela a vcs(tipo off-topic) por nao ter mais ninguem pra pedir ajuda.











Será q essa questao realmente dá pra fazer usando apenas uma integral?











Por favor quem poder me ajudar eu agradeço,





Roney Kevin











Yahoo! Mail - O melhor e-mail do
Brasil. Abra
sua conta agora!








[obm-l] RE: [obm-l] Trigonometria e Números Complexos

2004-03-22 Por tôpico Leandro Lacorte Recova









LINGUAJEM  ??? 



-Original Message-
From: [EMAIL PROTECTED]
[mailto:[EMAIL PROTECTED] On Behalf
Of Renato de Brito
Sent: Saturday, March 20, 2004
12:01 PM
To: [EMAIL PROTECTED]
Subject: [obm-l] Trigonometria e
Números Complexos





Vocês acham que o livro Trigonometria e Números
Complexos(Coleção do Professor de Matemática) esta em linguajem acessível a um
aluno do Ensino médio.










RE: [obm-l] Re: Duvida em somatorio

2004-03-18 Por tôpico Leandro Lacorte Recova
Gostei da palavra higienizado !!! 

Saudacoes Prof. Morgado,

Leandro. 

-Original Message-
From: [EMAIL PROTECTED] [mailto:[EMAIL PROTECTED] On
Behalf Of Augusto Cesar de Oliveira Morgado
Sent: Thursday, March 18, 2004 4:29 PM
To: [EMAIL PROTECTED]
Subject: Re: [obm-l] Re: Duvida em somatorio

Bote as constantes em evidencia, multiplique por 1-p, chame 1-p de x e o seu

problema, devidamente higienizado , passou a ser calcular F(x) = somatorio 
(x^n)/n, para x entre -1 e 1. Facilmente se ve que F'(x)= somatorio x^(n-1)
= 
1/(1-x). Logo, F(x)= 1 - ln (1-x)

==
Mensagem  enviada  pelo  CIP  WebMAIL  - Nova Geração - v. 2.1
CentroIn Internet Provider  http://www.centroin.com.br
Tel: (21) 2542-4849, (21) 2295-3331Fax: (21) 2295-2978
Empresa 100% Brasileira - Desde 1992 prestando servicos online


-- Original Message ---
From: niski [EMAIL PROTECTED]
To: 
Cc: [EMAIL PROTECTED]
Sent: Thu, 18 Mar 2004 20:20:18 -0300
Subject: [obm-l] Re: Duvida em somatorio

 Ah esqueci!! p é uma constante e esta no intervalo [0,1]
 
 niski wrote:
 
  Pessoal, alguem poderia mostrar como resolver esse somatorio por favor?
  (ele veio do calculo da esperança de 1/X onde X segue uma distribuicao 
  geometrica)
  
  Somatorio[n=1 , +inf] [(1/n)*p*(1-p)^(n-1)]
  
  Obrigado pessoal.
 
 
 -- 
 Niski - http://www.linux.ime.usp.br/~niski
 
 When we ask advice, we are usually looking for an accomplice.
 Joseph Louis LaGrange
 
 =
 Instruções para entrar na lista, sair da lista e usar a lista em
 http://www.mat.puc-rio.br/~nicolau/olimp/obm-l.html
 =
--- End of Original Message ---

=
Instruções para entrar na lista, sair da lista e usar a lista em
http://www.mat.puc-rio.br/~nicolau/olimp/obm-l.html
=

=
Instruções para entrar na lista, sair da lista e usar a lista em
http://www.mat.puc-rio.br/~nicolau/olimp/obm-l.html
=


[obm-l] RE: [obm-l] Dúvida - porcentagem

2003-03-14 Por tôpico Leandro Lacorte Recova
Caro Rodrigo,

Eu pensei assim: 

X - Preco de cada unidade
N1 - Numero de pecas vendidas antes de reduzir o preco.
Fat - Faturamento antes de reduzir o preco. Fat = N1*X

X2 - Preco de cada Unidade com reducao do preco de 20% - X2 = 0.8*X 
N2 - Numero de Pecas Vendidas com preco reduzido 
Fat2 - Faturamento com preco reduzido - Fat2 = N2*X2 

Porem pelos dados do problema temos Fat2 = 1.6Fat = N2*X2 = 1.6N1*X1 =
N2/N1 = 1.6 * X1/X2 = 1.6 * X1/0.8X1 = 2. 

Portanto, a opcao certa seria a letra (C). 

Se eu estiver errado alguem  me corrija. 

Leandro. 


-Original Message-
From: [EMAIL PROTECTED]
[mailto:[EMAIL PROTECTED] On Behalf Of Rodrigo I.
Sent: Friday, March 14, 2003 2:03 PM
To: [EMAIL PROTECTED]
Subject: [obm-l] Dúvida - porcentagem

Por favor alguém tem alguma resolução pra esse exercício:
(Unificado - Vunesp - 2003) Um fabricante de um produto estima que uma
redução de 20% no preço ao consumidor implicará um aumento de 60% no
faturamento. Verificadas estas condições, a quantidade vendida do
produto, em relação à situação anterior à redução de preços, deverá:

a) quadruplicar
b) triplicar
c) duplicar
d) permanecer constante
e) cair pela metade

Obrigado
Rodrigo
-- 
__
Sign-up for your own FREE Personalized E-mail at Mail.com
http://www.mail.com/?sr=signup


=
Instruções para entrar na lista, sair da lista e usar a lista em
http://www.mat.puc-rio.br/~nicolau/olimp/obm-l.html
O administrador desta lista é [EMAIL PROTECTED]

=
=
Instruções para entrar na lista, sair da lista e usar a lista em
http://www.mat.puc-rio.br/~nicolau/olimp/obm-l.html
O administrador desta lista é [EMAIL PROTECTED]
=